Board Study Questions

Ace your homework & exams now with Quizwiz!

1. mobilization of the talocrural joint. Author: Dutton M Title: Dutton's Orthopaedic Examination, Evaluation, and Intervention Edition: 4 Publisher: McGraw-Hill Year: 2017 Pages: 1139, 1142, 1172-1174 1. At 12 weeks post fracture the goals of the functional phase should be to restore joint kinematics and attain full range of pain-free motion. Joint mobilization should improve accessory motion, decrease guarding, and lengthen the tissue around a joint. (pp. 1139, 1172-1174) 2. Stretching the tibialis anterior will not help the limited ankle dorsiflexion but may improve plantar flexion. This should not be performed first. (pp. 1172-1174) 3. Before progressing to strengthening the gastrocnemius, emphasis should be on strengthening the dorsiflexors to address the chief complaint (pp. 1172-1174). 4. Catching of the toe, not foot drop, was the chief complaint. Use of an ankle-foot orthosis may be premature, because the patient may improve range of motion with mobilization and strengthening. (p. 1142)

1.1 A patient is 12 weeks post talus fracture and has a chief complaint of catching her toe during gait since the removal of the boot cast. Evaluation demonstrates limited ankle dorsiflexion, leg atrophy, and ankle weakness. The physical therapy plan should FIRST emphasize: 1. mobilization of the talocrural joint. 2. stretching the tibialis anterior. 3. strengthening the gastrocnemius. 4. fitting for an ankle-foot orthosis (AFO)

3. Apraxia 1. Horizontal nystagmus is a symptom of a cerebellar problem, such as a lesion of the anterior inferior cerebellar artery. 2. An ataxic gait is a symptom of a cerebellar problem, such as a lesion of the basilar artery. 3. Apraxia is a clinical symptom of a middle cerebral artery lesion. 4. Rigidity is not caused by a lesion of middle cerebral artery.

1.10 A patient had a ruptured right middle cerebral artery aneurysm that was repaired. Which of the following functional limitations would the patient MOST likely exhibit? 1. Horizontal nystagmus 2. Ataxic gait 3. Apraxia 4. Rigidity

1. Upper trapezius 1. The upper trapezius elevates the shoulder alone but, coupled with the lower trapezius and serratus anterior, produces upward rotation of the scapula via force coupling (p. 64). 2. The posterior deltoid extends, abducts, and laterally (externally) rotates the shoulder (p. 81). 3. The rhomboids retract, elevate, and downwardly rotate the scapula (p. 65). 4. The teres major extends, adducts, and medially (internally) rotates the shoulder (p. 77).

1.13A physical therapist is evaluating a patient who reports shoulder pain during overhead activities. During active shoulder abduction on the affected side, the patient demonstrates diminished scapular upward rotation. Weakness of which of the following muscles is MOST likely to contribute to this dysfunction? 1. Upper trapezius 2. Posterior deltoid 3. Rhomboids 4. Teres major

2. Have the patient use 2 axillary crutches. Author: O'Sullivan SB, Schmitz TJ, Fulk GD Title: Physical Rehabilitation Edition: 6 Publisher: F.A. Davis Year: 2014 Pages: 429, 464, 472, 475, 479, 6961. A walker would accommodate the weight-bearing restrictions but would be more restrictive than bilateral axillary crutches (pp. 475, 479). 2. Physical therapists should chose the least restrictive device that the patient can safely use. Given the patient is familiar with use of crutches, having the patient use crutches bilaterally would be most appropriate. (pp. 429, 696) Crutches are used to improve balance and to relieve weight-bearing fully or partially on a lower extremity. They are typically used bilaterally. (p. 472) 3. Bilateral axillary crutches are needed to unload the left lower extremity sufficiently. Single devices are not intended for use with restricted weight-bearing gait. (p. 472) 4. Canes are not intended for use with restricted weight-bearing gaits (p. 464

1.14 A patient with a left tibial fracture is restricted to 25% weight-bearing. The patient is currently walking with a single axillary crutch on the left side. Which of the following is the MOST appropriate action for the physical therapist? 1. Have the patient use a walker instead of a crutch. 2. Have the patient use 2 axillary crutches. 3. Switch the crutch to the patient's right side. 4. Prescribe a quad cane to use on the left side.

3. Compression of the shafts of the tibia and fibula at mid calf Author: Magee DJ Title: Orthopedic Physical Assessment Edition: 6 Publisher: Elsevier Saunders Year: 2014 Pages: 936 Author: Dutton M Title: Dutton's Orthopaedic Examination, Evaluation, and Intervention Edition: 4 Publisher: McGraw-Hill Year: 2017 Pages: 1085, 1131-11321. The talar tilt test with the ankle in neutral dorsiflexion is used for evaluating the integrity of the calcaneofibular ligament. 2. The anterior drawer test with the ankle in neutral dorsiflexion is used for evaluating the integrity of the anterior talofibular ligament. 3. Compression of the shafts of the tibia and fibula at mid calf is used to test for syndesmosis ligament injury, including injury to the anterior tibiofibular ligament. 4. Squeezing the calf with the ankle in neutral dorsiflexion is used to test the integrity of the Achilles tendon

1.17 To assess an anterior tibiofibular ligament injury, which of the following tests is MOST appropriate? 1. Talar tilt with the ankle in neutral dorsiflexion 2. Anterior drawer at the ankle with the ankle in neutral dorsiflexion 3. Compression of the shafts of the tibia and fibula at mid calf 4. Squeezing the calf with the ankle in neutral dorsiflexion

3. Aching fingers resulting from chronic osteoarthritis Author: Cameron MH Title: Physical Agents in Rehabilitation: From Research to Practice Edition: 4 Publisher: Elsevier Saunders Year: 2013 Pages: 7, 133, 150, 152-153, 1571. Applying heat may increase the edema due to vasodilation and increased metabolic rate, leading to an increase in inflammation (p. 152). Paraffin as a thermotherapy agent would also be difficult to remove from the healing site of the incision (p. 153). 2. Paraffin wax is used for thermotherapy (p. 157). Cryotherapy is usually recommended for chronic inflammatory conditions such as rheumatoid arthritis (p. 133). 3. As long as no active swelling is noted, paraffin will help increase motion and decrease pain associated with chronic osteoarthritis in the fingers (p. 150). 4. Thermotherapy, aside from the application of neutral warmth, may aggravate the pain associated with complex regional pain syndrome (p. 7).

1.18Paraffin would be MOST beneficial for a patient with which of the following conditions? 1. Edematous wrist 1 week following carpal tunnel surgery 2. Swollen elbow resulting from rheumatoid arthritis exacerbation 3. Aching fingers resulting from chronic osteoarthritis 4. Painful hand resulting from early-stage complex regional pain syndrome

1. Lower extremity partial squats Author: Andrews JR, Harrelson GL, Wilk KE Title: Physical Rehabilitation of the Injured Athlete Edition: 4 Publisher: Elsevier Saunders Year: 2012 Pages: 397-399, 401-4041. The photograph shows the anterior drawer test for the knee, which is used to assess the integrity of the anterior cruciate ligament. Closed kinetic chain extension exercises will put less stress on the anterior cruciate ligament and are appropriate for the subacute phase (week 4) of treatment. 2. Open kinetic chain knee extension, especially the last 25°, will put increased tension on the anterior cruciate ligament. 3. Although straight leg raises put no stress on the anterior cruciate ligament, this would be an exercise for the acute (not subacute) phase of treatment. 4. Plyometric exercises are an important part of the functional phase (week 10) of rehabilitation after an anterior cruciate ligament tear

1.19 A patient has a positive result on the anterior drawer test. During the subacute phase of treatment, the MOST appropriate intervention for the patient is independent performance of which of the following exercises? 1. Lower extremity partial squats 2. Open kinetic chain knee extension 3. Straight leg raises 4. Plyometric exercises

2. The rate is secondary to an increased stroke volume. Author: Kraemer WJ, Fleck SJ, Deschenes MR Title: Exercise Physiology: Integrating Theory and Application Edition: 1 Publisher: Lippincott, Williams & Wilkins Year: 2012 Pages: 140, 144-1461. It is more likely that the bradycardia is a training effect associated with a greater stroke volume (pp. 144-146). 2. Cardiac output is the product of stroke volume multiplied by heart rate. A training effect is an increase in stroke volume. There is a resultant decrease in heart rate to maintain the same cardiac output at rest. (pp. 144-146) 3. It is more likely that the bradycardia is a training effect associated with a greater stroke volume (pp. 144-146). 4. Exercise training increases parasympathetic activity and causes a small decrease in sympathetic discharge. Training also decreases the intrinsic firing rate of the sinoatrial node. These training adaptations explain the resting bradycardia in individuals who train aerobically. (p. 140)

1.21 The resting heart rate of a 32-year-old runner is measured at 46 bpm. Which of the following explanations for this heart rate is MOST likely? 1. The individual has a hypotensive disorder. 2. The rate is secondary to an increased stroke volume. 3. The individual has an atrioventricular block. 4. Endurance training has stimulated the sympathetic nervous system

3. Excess caffeine intake Author: Goodman CC, Fuller KS Title: Pathology: Implications for the Physical Therapist Edition: 4 Publisher: Elsevier Saunders Year: 2015 Pages: 538-5391. Palpitations can occur as a result of hormonal changes (i.e., during menopause or with ovulation). Since this is a healthy male patient, hormonal changes associated with gender can be ruled out. 2. Generally, a low-activity-level/nonathletic lifestyle does not cause or increase the likelihood of palpitations. Exercise can both induce and reduce the frequency and onset of palpitations. 3. Palpitations can occur due to diet, particularly with excessive intake of caffeine. Typically, caffeine intake precipitates the palpitations and causes brief palpitations of gradual onset and without any associated pain, dizziness, or light-headedness. 4. Typically, palpitations of cardiac origin are associated with dyspnea, fainting, or severe light-headedness or dizziness. This patient does not have any of these additional symptoms

1.25 A nonathletic male patient reports occasional brief palpitations that occur in the absence of pain, dizziness, or light-headedness. The patient has no personal or familial history of heart disease and is otherwise healthy. Which of the following factors is the MOST likely source of the palpitations? 1. Gender 2. Sedentary activity level 3. Excess caffeine intake 4. Cardiac abnormality

4. Presence of shortness of breath and dependent edema Author: Goodman CC, Snyder TEK Title: Differential Diagnosis for Physical Therapists: Screening for Referral Edition: 5 Publisher: Elsevier Saunders Year: 2013 Pages: 254-2571. An increase in fatigue is a symptom of heart failure exacerbation; blood pressure changes are not. 2. A productive cough is a symptom of heart failure; heart rate changes are not. 3. Cyanosis is a symptom of heart failure; diaphoresis is not. 4. Shortness of breath and dependent edema are symptoms of heart failure exacerbation

1.26 A patient admitted to the hospital with a diagnosis of exacerbation of heart failure is preparing for discharge to home. Which of the following clinical characteristics would be MOST important to monitor as part of the home program? 1. Blood pressure and fatigue level 2. Heart rate and cough productivity 3. Presence of cyanosis and diaphoresis 4. Presence of shortness of breath and dependent edema

3. decreased hip extension at terminal stance. Author: Tecklin JS Title: Pediatric Physical Therapy Edition: 5 Publisher: Lippincott, Williams & Wilkins Year: 2015 Pages: 555, 561 1. Children with juvenile rheumatoid arthritis ambulate with a decreased cadence. 2. Children with juvenile rheumatoid arthritis ambulate with decreased plantar flexion at toe off (preswing) and terminal stance. 3. Children with juvenile rheumatoid arthritis ambulate with decreased hip extension at terminal stance and toe off (preswing). 4. Children with juvenile rheumatoid arthritis ambulate with increased anterior pelvic tilt throughout the gait cycle

1.3 A 6-year-old patient with juvenile rheumatoid arthritis involving the cervical spine, bilateral hips, knees, and ankles is referred to the physical therapy department. The patient has developed contractures of all involved joints and continues to complain of morning stiffness. A gait deviation that the physical therapist is likely to observe is: 1. increased cadence. 2. increased plantar flexion range at toe-off (preswing). 3. decreased hip extension at terminal stance. 4. decreased anterior pelvic tilt throughout the gait cycle

2. Pacing 1. Incentive spirometry is used to improve inspiratory volumes and chest expansion. It should be used with caution in patients who have chronic obstructive pulmonary disease. It does not improve exercise tolerance. (Reid, p. 261) 2. Using pacing, the patient would learn to work within his or her exercise tolerance. This may mean walking slower or walking with breaks and would allow greater total walking distance without shortness of breath. (O'Sullivan, p. 513) 3. Diaphragmatic breathing is used to decrease the work of breathing and improve diaphragmatic movement, not to improve exercise tolerance (Frownfelter, pp. 357-358; Hillegass, pp. 550-551). 4. Segmental breathing is used for patients with chest hypomobility to augment localized lung expansion (Frownfelter, pp. 362-364). Chest hypomobility is not usually an issue in patients who have chronic obstructive pulmonary disease.

1.33 A patient who has chronic obstructive pulmonary disease becomes short of breath when walking 5 feet (1.5 m) with a rolling walker. Which of the following techniques would be MOST appropriate in order to increase the distance the patient is able to walk without becoming short of breath? 1. Incentive spirometry 2. Pacing 3. Diaphragmatic breathing 4. Segmental breathing

4. plastic basin filled with tap water and move the transducer approximately 0.25 in (0.17 cm) away from the body surface. Author: Bellew JW, Michlovitz SL, Nolan TP Jr Title: Michlovitz's Modalities for Therapeutic Intervention Edition: 6 Publisher: F.A. Davis Year: 2016 Pages: 99 Author: Cameron MH Title: Physical Agents in Rehabilitation: From Research to Practice Edition: 4 Publisher: Elsevier Saunders Year: 2013 Pages: 180, 187 1. A whirlpool will increase the intensity of ultrasound by reflecting waves. Optimally ultrasound wave reflection should be reduced, not increased. 2. A metal basin will increase the intensity of ultrasound by reflecting waves, which is incorrect. 3. The immersion technique requires holding the ultrasound head away from skin. Holding the transducer directly on the skin is incorrect. 4. This option has the correct basin (with reduced reflection) and application (parallel to and the correct distance from the body surface)

1.5 A physical therapist uses underwater ultrasound as part of the intervention to treat a patient with an ankle injury. The MOST appropriate mode of application is to immerse the patient's ankle in a: 1. whirlpool filled with degassed water and hold the transducer underwater directly on the skin. 2. metal basin filled with mineral oil and hold the transducer underwater approximately 1 in (2.54 cm) away from the body surface. 3. ceramic basin filled with glycerin and move transducer underwater directly on the skin. 4. plastic basin filled with tap water and move the transducer approximately 0.25 in (0.17 cm) away from the body surface

3. International normalized ratio Author: Goodman CC, Fuller KS Title: Pathology: Implications for the Physical Therapist Edition: 4 Publisher: Elsevier Saunders Year: 2015 Pages: 1712-1713, 17151. Hemoglobin would not be changed by anticoagulant medication. Hemoglobin values measure the oxygen-carrying capacity of the red blood cells. (p. 1713) 2. Red blood cell counts are not changed by anticoagulants. The red blood cell count is a method used to assess the oxygen-carrying capacity of the blood. (p. 1712) 3. Warfarin (Coumadin) is an anticoagulant. The physical therapist must be aware when a patient is taking an anticoagulant so that treatment can be modified if there is an increased risk of hemorrhage. The international normalized ratio (INR) was developed to provide results that would not vary between laboratories. Therapeutic anticoagulation requires an INR of 2 to 3. As the INR increases above these values, the risk of bleeding during activity is increased. (pp. 1712-1713) 4. The erythrocyte sedimentation rate is not expected to be affected by anticoagulant medication. The erythrocyte sedimentation rate is used to identify inflammatory or necrotic processes. (p. 1715)

1.50 Which of the following laboratory values should a physical therapist monitor when treating a patient who is taking warfarin (Coumadin)? 1. Hemoglobin 2. Red blood cell count 3. International normalized ratio 4. Erythrocyte sedimentation rate

4. Squeeze therapy putty between the sides of the fingers. Author: Kisner C, Colby LA Title: Therapeutic Exercise: Foundations and Techniques Edition: 6 Publisher: F.A. Davis Year: 2012 Pages: 378-3791. The muscles that are active during this movement are supplied by the median nerve, and impingement occurs in the carpal tunnel, not at the hamate (p. 378). 2. Doing this exercise would not isolate muscles supplied by the ulnar nerve, as only the 4th and 5th digits would be involved. This exercise would be appropriate if the ulnar nerve was entrapped at the level of the cubital tunnel and not in the tunnel of Guyon (at the level of the hamate). (p. 379) 3. This exercise would strengthen muscles supplied by the median nerve at the wrist. Deficits here would be loss of thumb (1st digit) abduction and opposition. (p. 378) 4. This movement isolates the lumbricals and interossei, which are innervated by the ulnar nerve and are affected when entrapment occurs at the tunnel of Guyon (at the level of the hamate) (p. 379)

1.7 A patient is referred to physical therapy with a history of ulnar nerve entrapment at the level of the hamate. Which of the following would be the MOST specific exercise to improve this patient's strength deficits? 1. Practice pinching between thumb (1st digit) and the tip of the index finger (2nd digit). 2. Squeeze hand grip with elastic-band resistance. 3. Oppose thumb (1st digit) to the metacarpal phalangeal joint of each finger (2nd through 5th digits). 4. Squeeze therapy putty between the sides of the fingers.

2. Atraumatic instability Author: Dutton M Title: Dutton's Orthopaedic Examination, Evaluation, and Intervention Edition: 4 Publisher: McGraw-Hill Year: 2017 Pages: 588, 665-666, 670 Author: Magee DJ Title: Orthopedic Physical Assessment Edition: 6 Publisher: Elsevier Saunders Year: 2014 Pages: 3181. The patient is too young for insidious onset of adhesive capsulitis, and full lateral (external) rotation can be achieved in this patient. With adhesive capsulitis, impaired range of motion would be expected. (Dutton, pp. 665-666) 2. Symptoms occur with excessive abduction and lateral (external) rotation of the shoulder. Anterior instability should be considered. (Dutton, p. 670) 3. Although the closed packed position for the acromioclavicular joint is at 90° of abduction, apprehension with lateral (external) rotation of the shoulder is not cited in any test for the acromioclavicular joint (Dutton, p. 588). 4. Labral tears are commonly associated with traumatic injury with sudden onset (Magee, p. 318)

1.8 A 14-year-old baseball player reports shoulder pain of insidious onset. The patient displays apprehension when the shoulder is passively positioned in abduction and full external (lateral) rotation. Which of the following pathologies is MOST likely present in this individual? 1. Adhesive capsulitis 2. Atraumatic instability 3. Acromioclavicular separation 4. Superior labral tear

.4. Gentle, sustained passive hip extension Author: Kisner C, Colby LA Title: Therapeutic Exercise: Foundations and Techniques Edition: 6 Publisher: F.A. Davis Year: 2012 Pages: 88, 94-95, 109, 7521. This exercise requires the addition of active relaxation of the hip flexors and active or passive movement into hip extension to be effective (pp. 94-95). 2. This exercise is a strengthening exercise for the hip extensors; active movement of tight muscles does not activate a relaxation response; an isometric contraction is required (p. 752). 3. Tight muscles need to be taken to their most lengthened position before maintaining the position; lying prone on pillows is not the most lengthened position of the hip flexors (p. 109). 4. Gentle, sustained passive hip extension is an appropriate method of stretching tight tissues (p. 88).

1.9Which of the following is the MOST appropriate technique to improve the flexibility of the hip flexors? 1. Active hip extension to end range, followed by isometric hip flexion 2. Resisted hip extension using cuff weights, followed by active hip flexion 3. Placing the patient in prone with pillows positioned under the abdomen 4. Gentle, sustained passive hip extension

4. 30% to 40% higher. 1. Arm exercise typically results in 30% to 40% lower maximal oxygen uptake than leg exercise

2.12 When compared to maximal oxygen uptake values obtained in a lower extremity exercise test, values obtained in an upper extremity exercise test are typically: 1. 30% to 40% lower. 2. the same. 3. 10% to 20% higher. 4. 30% to 40% higher

2. Upper trapezius and serratus anterior 1. The upper trapezius is a prime mover in scapular elevation and upward rotation. The rhomboids are prime movers in shoulder retraction and elevation as well as scapular downward rotation. These two muscles are not the best pair to promote a balanced force couple because they are pulling in different directions to accomplish different motions. (p. 136) 2. The upper trapezius muscle pulls up and the lower fibers of the serratus anterior pull outward in a horizontal direction. The net effect is that together the two muscles pull in different directions to accomplish the same motion, which is the scapula rotating upward. (pp. 135, 137-139) 3. The latissimus dorsi does not affect the movement of the scapula significantly. It works to extend, adduct, medially (internally) rotate, and hyperextend the shoulder. The lower trapezius depresses and upwardly rotates the scapula. This answer is incorrect because the two muscles together do not produce a balanced force couple. (pp. 135, 152) 4. Although the serratus anterior is involved in stabilization of upward rotation, the latissimus dorsi does not affect the movement of the scapula significantly (pp. 137, 152).

3.50 During initial examination of a patient who has rotator cuff tendinitis, a physical therapist notices that, during arm elevation, the involved scapula upwardly rotates early, compared to the uninvolved scapula. Passive range of motion of the involved shoulder is normal. Based on the findings, which of the following pairs of muscles BEST stabilizes the scapula to promote a balanced force couple on the involved side? 1. Rhomboids and upper trapezius 2. Upper trapezius and serratus anterior 3. Latissimus dorsi and lower trapezius 4. Latissimus dorsi and serratus anterior

2. Obtaining a toy when placed out of reach 1. Pulling to stand is too difficult for a short-term physical therapy goal for this child, who is functioning at a 5-month to 6-month level. This activity would require intact protective extension and trunk righting. (p. 61) 2. This child is at the second stage of sitting (around 5 months) and would benefit from activities that challenge trunk righting, sitting balance, and protective reactions. These are within the child's ability to achieve, because they are items in the third stage of sitting, which usually occurs around age 6-7 months. (pp. 56-57) 3. No position is noted. This task could be achieved in a supine position. Holding a toy within the center of mass in sitting position is also a skill that is likely already achieved in the second stage of sitting, at age 5-6 months. (p. 57) 4. Transitioning from sitting to quadruped position is a task that is generally achieved in the third quarter of the first year, so it is not an appropriate goal of short-term physical therapy for a child who is in the 5-month to 6-month range of development, which is consistent with second stage of sitting. (p. 61)

A 24-month-old child who has a genetic disorder and developmental delay is able to sit independently but can easily be displaced and demonstrates emerging protective reactions and slow trunk-righting reactions in sitting position. Which of the following functional activities is BEST for short-term physical therapy for the child? 1. Pulling up to standing position 2. Obtaining a toy when placed out of reach 3. Holding a toy at midline with both hands 4. Transitioning from sitting to quadruped position

2. Sunken fontanelle 1. Increased respiratory rate is a sign of severe dehydration (not decreased rate). 2. The fontanelle will be sunken in infants who are dehydrated. 3. Cold hands and feet are present in severe dehydration. 4. In severe dehydration, the infant would be unable to cry

A mother reports that her 6-month-old infant has had vomiting and diarrhea over the last 2 days. Which of the following signs would be the MOST accurate indication of severe dehydration? 1. Decreased respiratory rate 2. Sunken fontanelle 3. Warm hands and feet 4. Loud crying when touched

4. Dynamic balance activities 1. The stem describes characteristics of a child with fetal alcohol syndrome. Severe delays in walking are not characteristic of a child with fetal alcohol syndrome, and it is likely that a child of this age would be walking independently. Since facial and physical features of fetal alcohol syndrome may be subtle, the diagnosis may not be made until the preschool or school-age years when inattention, hyperactivity, and learning problems are more apparent. (Tecklin, p. 158) 2. The stem describes characteristics of a child with fetal alcohol syndrome. Oversensitivity to sensory input is not a characteristic associated with fetal alcohol syndrome. This is more characteristic of children with autism spectrum disorders. (Tecklin, pp. 406, 415-416) 3. The stem describes characteristics of a child with fetal alcohol syndrome. Spasticity is not associated with fetal alcohol syndrome. Fine motor dysfunction, visuomotor deficits, and weak grasp are characteristics of children with fetal alcohol syndrome. (Palisano, p. 426) 4. The stem describes characteristics of a child with fetal alcohol syndrome. Fine motor dysfunction, visuomotor deficits, balance problems, and weak grasp are characteristics of children with fetal alcohol syndrome. Balance activities would benefit the child. (Palisano, p. 426)

A 4-year-old child has maxillary hypoplasia, an elongated mid face, and a short, upturned nose. The child has a short attention span and poor growth. Which of the following interventions would be MOST appropriate for the child? 1. Gait training with a rolling walker 2. Sensory desensitization activities 3. Activities to inhibit spasticity 4. Dynamic balance activities

4. Teach the parents range of motion exercises and positioning, encourage play in prone and sitting positions, and order customized seating insert for a power wheeled mobility device. 1. Upper extremity strengthening is not realistic, considering the progressive weakness expected with this child. The skill level would be too high. Manual propulsion is not functional over the long term. 2. The literature supports that children who have spinal muscular atrophy and who do not develop sitting ability are unlikely to walk and will require power mobility. They may become independent in a power wheelchair by age 1-2 years. 3. These skills are too advanced. The literature supports that children who have spinal muscular atrophy and who do not develop sitting ability are unlikely to walk and will require power mobility. 4. These interventions are most appropriate to the patient's impairments and functional limitations. The literature supports that children who have spinal muscular atrophy and who do not develop sitting ability are unlikely to walk and will require power mobility

A 4-year-old child who received a diagnosis of spinal muscular atrophy at age 9 months is referred for home physical therapy. The child is unable to sit without upper extremity support but rolls independently. The child has bilateral hip and knee flexion contractures that make use of the child's standing frame uncomfortable. Which of the following interventions are MOST appropriate for physical therapy?1. Teach parents airway clearance techniques, encourage upper extremity strengthening to prepare for wheelchair self-propulsion, and switch to using a modified prone stander. 2. Teach parents lower extremity stretching and strengthening exercises, adapt the standing frame to accommodate contractures, and encourage supported walking. 3. Teach parents proper transfers, facilitate upright positioning in kneeling and standing positions, and refer to an orthopedist for serial casting to address contractures. 4. Teach the parents range of motion exercises and positioning, encourage play in prone and sitting positions, and order customized seating insert for a power wheeled mobility device.

4. Prolonged sitting 1. Spondylolisthesis is incorrect because it is usually found in gymnasts, weight lifters, and football linemen as a result of repetitive flexion and hyperextension forces. It may also present as radiating pain into the buttocks or posterior thigh. (Shultz, pp. 379-380) 2. Disc herniation is incorrect because it occurs with movements into flexion, rotation, side flexion, or extension and does not have an insidious onset. The patient would exhibit radicular pain into the leg and foot that increases with extension. (Magee, p. 641) 3. Golfing is incorrect because golf is a sport, and sports injuries are usually not insidious. The patient reports an insidious onset of symptoms. The mechanism of injury for a herniated nucleus pulposus is repetitive movements into flexion, rotation, side flexion, or extension. (Magee, p. 641) 4. Prolonged sitting is correct because the patient's lower back pain developed over time. The patient has impaired sitting alignment as a result of postural alignment impairments and possible environmental factors. Prolonged sitting may contribute to increased back pain through a gradual creep of tissues (Magee, p. 557)

A 60-year-old patient reports the insidious development of low back pain with no radicular symptoms. There is no history of trauma or significant medical history. The patient plays golf once a week and reports sitting at a desk for approximately 6 hours each weekday. The patient has poor posture and significant loss of lumbar extension range of motion. Which of the following factors is the MOST likely cause of the patient's decreased range of motion? 1. Spondylolisthesis 2. Disc herniation 3. Golfing 4. Prolonged sitting

1. Ask questions about the precipitating factors for the dizziness 1. Dizziness is associated with disorders in many different systems as well as drug reactions. A more specific definition will lead the physical therapist to which system should be investigated in more detail. 2. The problem of dizziness should be further clarified before going on to another area, such as screening for changes in sensation. 3. After taking a complete history, the therapist should then perform an upper quarter screening, which would include assessing deep tendon reflexes of the upper extremities. 4. For the patient with dizziness, the integrity of the vertebral artery should be tested during the specific tests and measures section of the examination, which follows the history and systems screening process

A patient being evaluated for a C4-C5 herniated disc also reports having periodic dizziness. Which of the following actions should the physical therapist perform INITIALLY? 1. Ask questions about the precipitating factors for the dizziness. 2. Ask questions about sensory changes. 3. Assess deep tendon reflexes of the upper extremities. 4. Assess the integrity of the vertebral artery system.

4. Small amplitude oscillations into tissue resistance at the limit of available joint motion in a posteroinferior direction 4. This mobilization technique is the most effective in increasing tissue flexibility in the absence of pain. The glide is in the direction that will maximize arthrokinematic motion. (p. 134)

A patient demonstrates active, pain-free shoulder range of motion that is moderately limited in horizontal adduction. Which of the following joint mobilization techniques is MOST appropriate? 1. Large amplitude oscillations performed at the beginning of the range of motion in an anteroinferior direction 2. Small amplitude oscillations into tissue resistance up to the limit of available motion in an anterosuperior direction 3. Large amplitude oscillations within the available range of motion in a posterior direction 4. Small amplitude oscillations into tissue resistance at the limit of available joint motion in a posteroinferior direction

2. Shortening of the gastrocnemius 2. A common cause of decreased dorsiflexion is a tight Achilles tendon or shortening of the gastrocnemius.

A patient exhibits heel rise in the midstance phase of gait. Which of the following impairments is MOST likely associated with this finding? 1. Shortening of the hamstrings 2. Shortening of the gastrocnemius 3. Weakness of the tibialis anterior 4. Weakness of the flexor digitorum longus

1. Swimming 1. High-impact sports are contraindicated for this patient. Hip positioning into end-range flexion, adduction, and medial (internal) rotation is also contraindicated. Freestyle swimming does not involve either of these two concerns. 2. Water skiing involves both high-impact activity and the possibility of positioning in end-range flexion, both of which are contraindicated for this patient. 3. Stationary rowing involves positioning in end-range flexion, which is contraindicated for this patient. 4. Playing racquetball involves high-impact activity, which is contraindicated for this patient.

A patient had a total hip arthroplasty with a constrained (cemented) prosthesis using a posterolateral approach. Which of the following recreational activities has the LEAST risk of causing prosthetic loosening or dislocation for this patient? 1. Distance freestyle swimming 2. Water skiing 3. Stationary rowing 4. Playing racquetball

3. Splints with wrists in neutral position 1. Strengthening can be difficult especially with pain of the acute phase. 2. Active exercise (needed for functional fine motor tasks) has questionable benefit in the acute phase. 3. Splints can be applied to rest the involved joints, prevent excessive movement, and reduce mechanical stresses, all of which are desired outcomes in the acute phase of rheumatoid arthritis. 4. Passive stretching exercises are important as a part of a rehabilitation effort; however, given the acute nature of the problem, rest and protection are paramount, making this option inappropriate.Author: Goodman CC, Fuller KS Title: Pathology: Implications for the Physical Therapist Edition: 4 Publisher: Elsevier Saunders Year: 2015 Pages: 1324-1327

A patient has acute rheumatoid arthritis involving the wrist joints. Which of the following interventions is MOST appropriate? 1. Resistive exercises to end range 2. Functional fine motor tasks 3. Splints with wrists in neutral position 4. Passive stretching exercises

4. Pressure off-loading 1. An ankle-brachial index of 0.68 indicates probable arterial insufficiency (p. 199). Elevation will decrease arterial flow and should not be used. 2. An ankle-brachial index of 0.68 indicates probable arterial insufficiency (p. 199). The wound description does not indicate any edema. Maintained compression would further compromise perfusion in the presence of arterial insufficiency (p. 199). 3. Hyperbaric oxygen can increase oxygen delivery to the wound; however, pressure unloading should occur first to remove the cause of the wound and prevent further breakdown (pp. 199, 362). 4. This wound is over a pressure area. In patients who have vascular disease, the skin and its supporting structures have a decreased ability to endure pressures. Therefore, patients who have vascular disease are at high risk for developing pressure injuries. Off-loading is a first response. (p. 129)

A patient has an ulcer on the right lateral malleolus. The ulcer has minimal drainage and measures 1.5 cm × 2.0 cm and 0.5 cm in depth. The patient has an ankle-brachial index of 0.68. Which of the following interventions should be undertaken FIRST? 1. Elevation 2. Compression 3. Hyperbaric oxygen 4. Pressure off-loading

3. Spirometry with segmental breathing exercises 1. Percussion and vibration are appropriate for airway clearance (pp. 315-316). However, the absence of adventitious breath sounds suggests the airway is clear. 2. Percussion and vibration with a nebulizer that delivers medication are appropriate for airway clearance (pp. 315-316). However, the absence of adventitious breath sounds suggests the airway is clear. 3. Adventitious breath sounds are abnormal sounds such as crackles, wheezes, and stridor. Atelectasis is a collapse of lung tissue. Spirometry and segmental breathing exercises are appropriate interventions for atelectasis without the presence of adventitious breath sounds (pp. 363-364, 496). 4. Incisional splinting is appropriate for pain control when coughing after abdominal surgery but would not address the atelectasis (p. 498)

A patient has right lower lobe atelectasis following abdominal surgery. Upon auscultation, a physical therapist notes no adventitious breath sounds. Which of the following is MOST appropriate for this patient? 1. Percussion and vibration in left Trendelenburg position 2. Vigorous percussion and vibration with nebulizer 3. Spirometry with segmental breathing exercises 4. Practice of incisional splinting for coughing techniques

1. Increased latency and decreased nerve conductive velocity 1. The clinical examination suggests compromise of the median nerve. Latency would be increased and would be inversely proportional to conduction velocity.

A patient has sensory deficits to pin-prick and muscle weakness in the median nerve distribution. Based on these findings, which of the following should be expected with testing of median nerve conduction velocity and latency? 1. Increased latency and decreased nerve conductive velocity 2. Increased latency and increased nerve conductive velocity 3. Decreased latency and decreased nerve conductive velocity 4. Decreased latency and increased nerve conductive velocity

2. Passive shoulder range of motion measures 1. Although date of onset is important information, it alone does not give much insight. For example, date of onset will not differentiate between adhesive capsulitis and cuff tear. 2. With just the information listed in the stem, it is impossible to determine whether the problem is adhesive capsulitis or rotator cuff tendinitis/tear, etc. Passive range of motion measures are needed to make this determination, i.e., full passive range of motion points to tendinitis, whereas limited passive range of motion points to adhesive capsulitis. 3. Deep tendon reflexes or neurological status would not be most useful in determining the source of the symptoms. 4. Although manual muscle testing scores for the rotator cuff are also important information, simple manual muscle testing of the cuff might not differentiate tendinitis from other problems such as adhesive capsulitis

A patient has shoulder pain and decreased shoulder active range of motion. The patient's past medical history is unremarkable. Which of the following information would be MOST helpful in determining the source of the patient's symptoms? 1. Date of onset of the problems 2. Passive shoulder range of motion measures 3. Upper extremity deep tendon reflex measures 4. Manual muscle testing scores for the rotator cuff

2. Use of a dynamic splint worn multiple hours throughout the day 1. Stretching exercises of short duration would produce changes over time through remodeling, but these changes would take much longer to occur. Creep, on the other hand, would produce lasting changes over a shorter period of time. (pp. 83-84) 2. This option uses the concept of creep. Creep is permanent deformation of tissue through the application of a low magnitude load over a long period of time (p. 84). The use of a dynamic splint will allow the application of such a load and induce rapid changes over an extended period with minimal tissue damage and inflammation (p. 89). 3. Ballistic stretches are not recommended for individuals after immobilization because they are thought to create significant trauma to atrophied tissues (pp. 89, 92). 4. Range of motion exercises are not thought to increase tissue length because they are not sustained long enough to induce lasting changes. They may be used to maintain tissue length but generally do not provide enough stress by themselves to increase tissue length. (p. 52)

A patient has significant shortening of tissues around a joint after immobilization. Patient instruction in which of the following interventions would BEST produce rapid, lasting changes in tissue length? 1. Stretching exercises of short duration performed multiple times throughout the day 2. Use of a dynamic splint worn multiple hours throughout the day 3. Stretching exercises of ballistic quality performed multiple times throughout the day 4. Multiple repetitions of range of motion exercises performed to end range

4. Anterior talofibular ligament 1. The photograph shows the anterior drawer test of the ankle (Magee, pp. 932-933). The Thompson (Simmonds) test is used to assess for tears of the Achilles tendon (Magee, p. 940). 2. The photograph shows the anterior drawer test of the ankle (Magee, pp. 932-933). Palpation of the proximal (base) fifth metatarsal is used to rule out avulsion fracture from a fibularis (peroneus) brevis tendon pull (Giangarra, p. 257). 3. The photograph shows the anterior drawer test of the ankle (Magee, pp. 932-933). The talar tilt test is used to determine whether the calcaneofibular ligament is torn (Magee, pp. 936-937). 4. The photograph shows the anterior drawer test of the ankle (Magee, pp. 932-933). A positive result may be obtained on the anterior drawer test only if the anterior talofibular ligament is torn; however, anterior translation is greater if both the anterior talofibular ligament and calcaneofibular ligaments are torn (Magee, pp. 932-933).

A patient has significant swelling around the lateral ankle. Radiographs of the ankle reveal no evidence of bone injury. An ant drawer test has a positive result. Which of the following structures is MOST likely injured? 1. Achilles tendon 2. Fibularis (peroneus) brevis tendon 3. Calcaneofibular ligament 4. Anterior talofibular ligament

4. Weak knee extensors 1. An excessively supinated foot occurs during the stance phase and would be accompanied by a general medial (internal) rotation of the lower extremity during stance, which is not evident in the photograph (p. 1005). 2. Decreased dorsiflexion strength will produce an ankle that remains plantar flexed during the swing phase of gait associated with dragging of the toes. The patient would have to hike the hip or circumduct the affected leg to decrease the toe drag. These gait deviations are not shown in the photograph. (p. 1005) 3. Increased limb length would result in the patient having to vault over the longer leg, circumduct the longer leg during swing phase, or hike the hip of the longer leg during swing phase. These gait deviations are not shown in the photograph. (pp. 1006-1007) 4. A patient who has weak knee extensors will likely exhibit genu recurvatum during the stance phase, as shown in the photograph (p. 1003)

A patient has the gait deviation shown in the photograph. Which of the following pathological conditions of the involved limb is MOST likely the cause? 1. Excessively supinated foot 2. Decreased ankle dorsiflexion strength 3. Increased limb length 4. Weak knee extensors

1. Pursed-lip breathing training 1. Pursed-lip breathing is not possible when the patient is intubated. 2. Diaphragmatic breathing is possible and helpful for a patient who has synchronized intermittent mandatory ventilation in ventilator mode, in which the patient is allowed to breathe spontaneously between machine-delivered breaths. 3. Deep breathing exercises are possible and helpful for a patient who has synchronized intermittent mandatory ventilation in ventilator mode, in which the patient is allowed to breathe spontaneously between machine-delivered breaths. 4. Lateral costal breathing is possible and helpful for a patient who has synchronized intermittent mandatory ventilation in ventilator mode, in which the patient is allowed to breathe spontaneously between machine-delivered breaths

A patient in an intensive care unit is intubated and is being treated with a mechanical ventilator. The patient would be UNABLE to participate in which of the following interventions? 1. Pursed-lip breathing training 2. Diaphragmatic breathing training 3. Deep breathing exercises 4. Lateral costal expansion exercises

3. Place a pillow roll under the patient's knees. 1. This option is not correct because manual stabilization of the pelvis is not indicated during upper abdominal muscle testing. Correct pelvic positioning should be achieved first before conducting the test. (pp. 199-201) 2. Placing a hand at the posterior lumbar spine would be done if trunk flexion was being tested by using the leg lowering method (p. 202). 3. The patient's short hip flexors are preventing posterior pelvic tilt in the supine position; placing a pillow roll under the knees will passively flex the hips and allow the lumbar spine to flatten in preparation for the upper abdominal muscle test. The first step in this muscle test is correct positioning of the patient. (p. 199) 4. Stabilization of the lower extremities should not be done because it increases activity in the hip flexors, allowing them to substitute for weak abdominals (p. 201)

A patient in supine position with lower extremities extended exhibits an anterior pelvic tilt. To test the patient's upper abdominal strength effectively, which of the following procedures should be implemented FIRST? 1. Manually stabilize the pelvis. 2. Place a hand at the posterior lumbar spine. 3. Place a pillow roll under the patient's knees. 4. Manually stabilize the lower extremities

1. Dynamic standing balance 1. Dynamic standing balance is a critical skill for independence with standing activities of daily living. The patient may not have adequate balance for standing hands free for dressing, etc. (Umphred, pp. 664-665) 2. The role of the vestibular system is to maintain clear vision during head motion (Umphred, p. 689). Vestibular function would affect activities of daily living in both positions. 3. The patient is able to perform movements involved in putting on pants in sitting position without difficulty. Therefore, sitting balance is not an issue with this patient, and an assessment of dynamic standing balance would be more important. (Umphred, pp. 664-665) 4. Range of motion in lower extremities would affect activities of daily living in both positions (Dutton, p. 521-522).

A patient is able to put on a pair of pants independently in sitting position but is unable to perform the activity in a standing position. Which of the following parts of the physical therapy examination is MOST important to identify the source of the patient's problem? 1. Dynamic standing balance 2. Vestibular function 3. Dynamic sitting balance 4. Lower extremity range of motion

3. Sleep in left sidelying and use pillows to elevate the head. 1. Frequent position changes and using sleeping positions that increase reflux (right sidelying and supine) will not improve the patient's ability to sleep through the night. 2. The lower esophagus bends to the left; lying on the right side straightens out the esophagus, increasing reflux. 3. For nocturnal reflux, the patient should be encouraged to sleep on the left side with a pillow in place to maintain this position. The lower esophagus bends to the left, so reflux is minimized in this position. 4. Although sleeping while sitting up might be an ultimate solution if the patient is unable to avoid nocturnal reflux, it should not be the first recommendation

A patient is frequently awakened at night by symptoms of gastroesophageal reflux disease. Which of the following recommendations should a physical therapist make FIRST? 1. Change sleeping positions every 2 hours. 2. Sleep in right sidelying and use pillows to elevate the head. 3. Sleep in left sidelying and use pillows to elevate the head. 4. Sleep sitting up in a comfortable chair

4. Myositis ossificans 1. Avascular necrosis of the femur causes hip pain due the lack of blood to the femoral head. There is usually an insidious onset without trauma to the quadriceps. (p. 256) 2. Osteochondritis dissecans is a necrotic bone lesion with no known cause. The knee, talus, and elbow can be involved. (p. 1148) 3. Slipped capital femoral epiphysis occurs when the capital femoral epiphysis becomes displaced. The patient is usually an adolescent male with limited hip range of motion and hip and thigh pain. This condition is not associated with quadriceps contusion. It is characterized by a sudden or gradual displacement of the femoral neck from the capital femoral epiphysis, while the head remains in the acetabulum. (p. 1580) 4. Myositis ossificans is a complication of quadriceps contusion and is caused by heterotropic bone formation on the femur. During quadriceps contraction, the muscle belly rubs across the bone, causing the sharp pain. (p. 767)

A patient is referred to physical therapy after a quadriceps contusion. After good initial progress with rehabilitation, the patient reports sharp pain of the anterior thigh and progressive loss of knee flexion. Which of the following conditions is MOST likely present? 1. Avascular necrosis of the femur 2. Osteochondritis dissecans 3. Slipped capital femoral epiphysis 4. Myositis ossificans

2. Extensor carpi radialis brevis 1. The supinator may be involved with radial tunnel syndrome and have a referral to the elbow. The radial nerve may refer pain to the lateral elbow. Evaluation should note possible sensory paresthesias in the superficial radial nerve distribution to rule out radial tunnel syndrome. However, the extensor carpi radialis brevis is the muscle most commonly affected. (Dutton, p. 764) 2. Lateral epicondylalgia (tennis elbow) is defined as a pathologic condition of the wrist extensor muscles at their origin on the lateral humeral epicondyle. The tendinous origin of the extensor carpi radialis brevis is the area of most pathologic change. Changes can also be found in the musculotendinous structure of the extensor digitorum communis. (Kisner, p. 636; Magee, p. 407) 3. The brachioradialis can refer symptoms to the elbow region but is not commonly linked to repetitive extension activities such as tennis (Hislop, p. 139). The brachioradialis acts as a shunt muscle, overcoming centrifugal forces acting on the elbow and also adds power to increase the speed of flexion (Dutton, p. 716). 4. Tendinitis of the triceps distal tendon at the olecranon may elicit some posterior elbow or olecranon pain (Dutton, p. 754), but the extensor carpi radialis brevis is the muscle most commonly associated with lateral elbow pain related to repetitive activity (Dutton, p. 764)

A patient reports lateral elbow pain after playing tennis for 2 hours. Which of the following muscles is MOST likely involved? 1. Supinator 2. Extensor carpi radialis brevis 3. Brachioradialis 4. Triceps

3. Schedule physical therapy for 1 hour after antiparkinsonian medications are given.

A patient with Parkinson disease exhibits typical end-of-dose akinesia during exercise. Which of the following courses of action is MOST appropriate for a physical therapist? 1. Review with patient and family the importance of adherence to medication doses. 2. Modify the exercise program to be less aggressive. 3. Schedule physical therapy for 1 hour after antiparkinsonian medications are given. 4. Discontinue physical therapy sessions until medications are modified.

3. is not reproduced during range of motion of the spine and hips. 1. Constant pain that is not dependent on position or activity and is increased with weight-bearing is a warning symptom (Goodman, p. 1255). Pain that subsides with movement or varies with intensity of movement can generally be considered to be of musculoskeletal origin. 2. Pain that awakens a patient from sleep, rather than pain that makes it difficult to fall asleep or pain upon waking, is considered a warning symptom (Goodman, p. 1255). Pain in the morning may be due to prolonged lying in a static posture. Morning stiffness or pain is common in patients who have low back pain (Magee, p. 558). 3. Pain arising from the spine is almost always influenced by posture and movement (Magee, p. 559). Therefore, pain not reproduced with range of motion testing of the back and hips would be cause for concern. 4. Prone positioning can extend the lumbar spine (Magee, p. 572). Therefore, pain abolished in this position would be considered mechanical or musculoskeletal in nature

A patient reports of back pain. The physical therapist should suspect that the pain is of a non-mechanical origin if the pain: 1. increases in intensity with active forward bending of the trunk. 2. is worse in the morning. 3. is not reproduced during range of motion of the spine and hips. 4. resolves completely when the patient lies prone.

1. Closed chain quadriceps strengthening exercises 1. A review of the literature showed strong evidence that both open and closed kinetic chain exercises are at least equally effective in reducing pain and improving function. In one study there appeared to be only a slight advantage to using only closed kinetic chain exercises over using open kinetic chain exercises. Both methods resulted in improved function and decreased pain up to 3 months after concluding the exercise program. 2. Plyometric exercises would apply too much stress on the anterior and lateral aspects of the knee, making symptoms worse. 3. Continuous ultrasound may be of benefit to improve elasticity of structures secondary to the deep heating effect; however, pulsed ultrasound does not produce a heating effect. 4. Patellar mobilization should be used to stretch lateral structures, not medial structures

A patient reports pain at the anterior and lateral aspects of the knee after prolonged sitting and when ascending or descending stairs. After applying tape to the knee, the patient is able to ascend and descend stairs without pain. Which of the following interventions is MOST appropriate to use in addition to the taping? 1. Closed chain quadriceps strengthening exercises 2. Plyometric exercises 3. Pulsed ultrasound to the lateral retinaculum 4. Patellar mobilization focusing on stretching medial structures

3. Stretch the right hip flexor muscles. 1. Diverticulitis would create left-sided, not right-sided, abdominal pain upon palpation (Goodman & Snyder, pp. 339-340). 2. The test position described in the question is for the iliopsoas muscle. Rectus abdominis symptoms with palpation would be more centered in the abdomen; therefore, there is no indication that the rectus abdominis muscle should be evaluated. (Goodman & Snyder, p. 333) 3. This is the test position to identify dysfunction of the iliopsoas muscles; therefore, stretching the right hip flexor, which is the muscle action of the iliopsoas, is the appropriate action (Goodman & Snyder, p. 333). 4. Thrombosis that occludes the iliac vein produces posterior lower leg swelling, pain or tenderness, dilation of superficial veins, and pitting edema. The patient does not have these signs or symptoms. (Goodman & Fuller, p. 649)

A patient reports right lower quadrant pain. When the patient is in the supine position with hips and knees flexed 90°, deep palpation in the right lower quadrant reproduces the pain. What is the MOST appropriate action for the physical therapist to do NEXT? 1. Inform the patient about possible diverticulitis. 2. Test rectus abdominis strength. 3. Stretch the right hip flexor muscles. 4. Contact the physician about possible iliac vein thrombosis

1. Lateral (external) rotation with abduction 1. Numbness and tingling over the thumb and index finger (1st and 2nd digits) involves the median nerve. Shoulder lateral (external) rotation with abduction is used to test the median nerve (upper limb tension test [ULTT 2a]). Shoulder lateral (external) rotation is added to 90° of shoulder abduction combined with shoulder girdle depression to place tension on the median nerve. 2. Numbness and tingling over the thumb and index finger (1st and 2nd digits) involves the median nerve. Shoulder medial (internal) rotation is used when testing for radial nerve involvement (upper limb tension test [ULTT 2b]). 3. Numbness and tingling over the thumb and index finger (1st and 2nd digits) involves the median nerve. Adduction of the shoulder would reduce tension on the median nerve; abduction increases tension on the nerve. 4. Numbness and tingling over the thumb and index finger (1st and 2nd digits) involves the median nerve. Medial (internal) rotation with adduction is not used to test upper extremity neural tension because it does not create adequate neural tension

A patient reports upper extremity numbness and tingling that extends from the neck to the thumb and index finger (1st and 2nd digits). Which of the following shoulder positions would MOST likely exacerbate the patient's symptoms? 1. Lateral (external) rotation with abduction 2. Medial (internal) rotation with abduction 3. Lateral (external) rotation with adduction 4. Medial (internal) rotation with adduction

4. Ability to walk but difficulty with distal upper extremity and hand function 1. The lower extremities are often spared with a central cord injury; therefore, a motorized wheelchair is an inappropriate assistive device. 2. The patient is more likely to have distal arm weakness, which should affect wheelchair mobility with upper extremities, but the patient may recover the ability to walk. 3. Patients who have central cord syndrome should expect to have difficulty with activities of daily living that require upper extremity function. Since the lower extremities may be spared, the patient might be able to transfer without a sliding board. Because the question asks for the highest level hoped for, this would not be the best choice. 4. This is a description of central cord syndrome. Because the spinal tracts for the lower extremities are positioned more laterally in the spinal cord, upper extremities are more affected than lower extremities. Patients with central cord syndrome typically recover the ability to ambulate with some remaining distal arm weakness. Seventy-seven percent of patients with central cord syndrome will attain ambulatory function, and 42% will attain hand function.

A patient sustained a cervical hyperextension injury that caused bleeding into the central gray matter of the lower cervical spinal cord. Which of the following descriptions BEST reflects the highest level of function the patient is likely to achieve? 1. Inability to transfer without assistance and use of a motorized wheelchair required for mobility 2. Ability to propel a standard wheelchair independently with upper extremities but inability to walk 3. Independence in bed mobility and self-care activities but use of a sliding board required for transfers 4. Ability to walk but difficulty with distal upper extremity and hand function

1. pressure garments. 1. Pressure garments worn 23 hours/day have been found to reduce the formation or thickness of hypertrophic scar (Bryant, p. 77). 2. Positioning may be used to maintain the soft tissue in a stretched position to counter scar contraction, but this would not affect the thickness of the scar tissue. 3. A walking exercise program can be helpful for regaining strength and endurance and would apply some stretch to scar tissue but would not provide the compression necessary to prevent an increase in the thickness of the tissue (hypertrophic scar formation). Directed functional exercise is the more appropriate approach (Sussman, p. 426) 4. It is important to maintain the soft tissue in a stretched position to counter scar contraction, but this would not affect the thickness of the scar tissue (Sussman, pp. 419, 426

A patient sustains severe chemical burns on both legs, requiring split-thickness skin grafts. To prevent hypertrophic scar formation, the BEST intervention is: 1. pressure garments. 2. functional positioning for the legs. 3. a walking exercise program. 4. prolonged muscle stretching.

3. Orthostatic hypotension 1. Angina is cardiac-related chest pain due to ischemia; it is unrelated to ringing in the ears (p. 534). 2. The hallmark sign of deep vein thrombosis is rapid onset of unilateral leg swelling, erythema, and heat (p. 901). 3. Orthostatic hypotension is a common complication in patients who have an acute cervical injury. It manifests with dizziness or light-headedness and ringing in the ears when in a vertical position, such as sitting or standing. (pp. 72, 896) 4. Autonomic dysreflexia is common in patients who have a spinal cord injury at T6 or above and results in significantly increased blood pressure and a pounding headache (p. 895)

A patient who has a C5 spinal cord injury (ASIA Impairment Scale A) suddenly reports light-headedness and ringing in the ears while sitting upright in a wheelchair. Which of the following conditions is MOST likely present? 1. Angina pectoris 2. Deep vein thrombosis 3. Orthostatic hypotension 4. Autonomic dysreflexia

4. Palpation of the radial portion of the thumb (1st digit) metacarpophalangeal joint 1. Passive range of motion into thumb (1st digit) metacarpophalangeal joint flexion should not stretch the involved ligament (p. 431). 2. Valgus stress testing of the metacarpophalangeal joint does not stress the injured side (p. 431). 3. Resisted isometric testing of the abductor pollicis longus should not stretch the affected ligament (p. 431). 4. The radial portion of the thumb (1st digit) metacarpophalangeal joint is the affected structure and would be tender to palpation (p. 32).

A patient who has a grade III ligamentous sprain of the radial collateral ligament of the metacarpophalangeal joint of the thumb (1st digit) is MOST likely to exhibit pain during which of the following tests? 1. Passive range of motion into thumb (1st digit) metacarpophalangeal joint flexion 2. Valgus stress testing of the metacarpophalangeal joint 3. Resisted isometric testing of the abductor pollicis longus 4. Palpation of the radial portion of the thumb (1st digit) metacarpophalangeal joint

2. Posterior capsule tightness 1. Functional horizontal adduction is performed in sitting or standing position, and strength testing for horizontal adduction is performed with the patient in supine position (Hislop, p. 128). Functional activities require accompanying movements at the scapula. The procedure shown in the photographs prevents scapular movement. 2. The photograph shows the technique to test for posterior capsular tightness. Retracting the scapula as illustrated removes the confounding compensatory effect of scapular protraction and allows isolation of the glenohumeral joint. The posterior capsule has been implicated as the source of restriction in this test (Magee, p. 285). In addition, patients who have diabetes are likely to experience shoulder disorders/mobility limitations (Goodman, p. 429). 3. Horizontal adduction does indeed test for pathological conditions of the acromioclavicular joint. However, the test is performed in sitting or standing position, and the scapula is not retracted, because one would need to be able to adduct the humerus to end range to elicit acromioclavicular joint symptoms. Retracting the scapula will limit horizontal adduction. (Magee, pp. 285, 330) 4. Assessment of scapular dyskinesia would require assessment of scapular mobility. The procedure shown in the photographs prohibits movement of the scapula. (Magee, p. 260).

A patient who has diabetes mellitus reports a progressive loss of shoulder mobility. A physical therapist performs the test shown in photographs A and B. (pt sidelying. Scapular mobility and arm rotation).Which of the following conditions is MOST likely being assessed? 1. Functional horizontal adduction 2. Posterior capsule tightness 3. Acromioclavicular joint tightness 4. Scapular dyskinesia

2. Muscle ache 1. Ptosis is not a common symptom of hypothyroidism. 2. Muscle ache (myalgia) is a common musculoskeletal symptom of hypothyroidism. 3. Dysphagia is not a common symptom of hypothyroidism. 4. Bradycardia, not tachycardia, is a common symptom of hypothyroidism

A patient who has hypothyroidism is MOST likely to exhibit which of the following signs or symptoms? 1. Ptosis 2. Muscle ache 3. Dysphagia 4. Tachycardia

3. Posterior pelvic tilts 1. The abdominals help to posteriorly tilt the pelvis, not anteriorly tilt the pelvis (p. 199). 2. This exercise is too advanced for someone with poor strength; the patient would not be able to move against gravity (pp. 200-201). Fair plus (3+/5) strength would be needed for this activity. 3. Posterior pelvic tilts incorporate an isometric contraction of the abdominals and would be helpful for strengthening initially (p. 199). 4. Oblique sit-ups require movement against gravity, which cannot be done with poor abdominal strength (pp. 200-201)

A patient who has low back pain exhibits Poor (2/5) strength of the lower abdominal muscles. Which of the following exercises would be most appropriate for the patient INITIALLY? 1. Anterior pelvic tilts 2. Partial sit-ups 3. Posterior pelvic tilts 4. Oblique sit-ups

2. Myocardial infarction 1. A premature atrial complex is indicated by an R wave that is close to the preceding R wave (p. 165). 2. A prominent, pathological Q wave is indicative of a transmural myocardial infarction (p. 170). 3. Supraventricular tachycardia is indicated by a diminished P wave, but the duration of the QRS complexes occur within an appropriate interval (p. 166). 4. Atrial fibrillation is characterized by inconsistent, irregular R-R intervals (p. 166)

A patient's electrocardiogram report describes the presence of significant Q waves. This finding is suggestive of which of the following conditions? 1. Premature atrial complex 2. Myocardial infarction 3. Supraventricular tachycardia 4. Atrial fibrillation

4. Sensation of the lateral aspect of the thigh 1. The adductor longus is innervated by the obturator nerve, and strength testing would assess the motor integrity of this nerve. Meralgia paresthetica does not involve the obturator nerve. 2. The quadriceps femoris is innervated by the femoral nerve, and strength testing would assess the motor integrity of this nerve. Meralgia paresthetica does not involve the femoral nerve. 3. Meralgia paresthetica is an entrapment or injury to the lateral femoral cutaneous nerve, a purely sensory nerve. Injury affects sensation to the lateral thigh. Sensory testing of the superior medial aspect of the thigh would be an assessment of the ilioinguinal nerve. 4. Meralgia paresthetica is an entrapment or injury to the lateral femoral cutaneous nerve, a purely sensory nerve. Injury affects sensation to the lateral thigh. Sensory testing of this region is the most appropriate assessment.

A patient who has meralgia paresthetica has been referred to physical therapy. Which of the following clinical features is MOST likely to be assessed by the physical therapist during the examination? 1. Strength of the adductor longus 2. Strength of the quadriceps femoris 3. Sensation of the superior medial aspect of the thigh 4. Sensation of the lateral aspect of the thigh

2. Posterior lower ribs 1. Segmental breathing combines breathing control with manual cues to specific areas of the chest wall. The lateral costal area would be a position to treat the lateral basal segments of the lower lobes, not the posterior basal segments. 2. Segmental breathing combines breathing control with manual cues to specific areas of the chest wall. Proper hand placement to encourage posterior basal expansion is over the posterior aspect of the lower ribs. 3. Segmental breathing combines breathing control with manual cues to specific areas of the chest wall. Midchest would be used to treat the superior segments of the lower lobe, not the posterior basal segments. 4. Segmental breathing combines breathing control with manual cues to specific areas of the chest wall. The anterior lower ribs would be a position to treat the anterior basal segments of the lower lobes, not the posterior basal segments

A patient who has pleural effusion is performing segmental breathing exercises. Where should manual counterpressure be applied to encourage expansion of the posterior basal segments of the patient's lower lobes? 1. Lower lateral costal area 2. Posterior lower ribs 3. Anterior midchest 4. Anterior lower ribs

3. Septic arthritis and ectopic bone formation 1. Actinomycosis has associated signs and symptoms of chest pain, dyspnea, fatigue, and fever (Ferri, p. 14). Osteomalacia is characterized by painful muscle weakness, bone pain, and tenderness (Goodman, pp. 1227-1228). Alkaline phosphatase levels are elevated. Since the patient is not experiencing chest pain, this is not the correct answer. 2. Alkaline phosphatase is an enzyme produced by bone cells. Increased levels would increase bone formation due to active osteoclasts (Goodman, p. 1232). This does not occur with a pyogenic infection or cellulitis. Cellulitis is associated with redness, tenderness, and edema (Goodman, pp. 700-701). 3. Septic arthritis is characterized by rapid onset, over hours or days, of monoarthritis with a joint that is swollen, red, tender, and warm with limited range of motion due to pain (Goodman, p. 1245). Increases in alkaline phosphatase levels with local edema, heat, and erythema are indicative of heterotopic ossification (ectopic bone formation) (Umphred, pp. 476-478; Goodman, pp. 1288-1289). 4. Neither granulomatous infection nor thrombophlebitis are associated with decreased glenohumeral range of motion (Goodman, pp. 230, 646)

A patient who has tetraplegia experiences an insidious onset of pain inferior to the glenohumeral joint. Soft tissue swelling, local warmth, and erythema are present, as well as limited range of motion. The patient's serum alkaline phosphatase levels are elevated. Which of the following conditions are MOST likely present? 1. Actinomycosis and osteomalacia 2. Pyogenic infection and cellulitis 3. Septic arthritis and ectopic bone formation 4. Granulomatous infection and thrombophlebitis

4. Vibration and percussion testing 1. Exertional compartment syndrome presents with subjective symptoms, including exertional leg pain (not the pin-point tenderness as presented in this case), decreased sensation after exertion, and paresthesias (Magee, p. 856). 2. Although these measures should be included in the examination, joint range of motion is usually maintained in persons with a stress fracture and would not be of utmost importance to include in the examination (Magee, p. 856). 3. Although gait assessment may be included as an examination measure, testing for conditions affecting the bones would be inherent at this point, because the patient is having pain with walking. The patient's gait may be altered due to pain at heel strike (initial contact), and an assessment at this point would not reflect the patient's normal gait. (Magee, p. 856) 4. Most individuals with stress fractures report an insidious onset of pain that correlates with a change in equipment or training and is exacerbated by the offending activity. They will have localized bony tenderness and palpable periosteal thickening, especially if they have long-standing symptoms. Some persons have pain at the fracture site with percussion or vibration at a distance from the fracture. Joint range of motion is usually maintained. Persons with a tibial stress fracture will have pin-point tenderness on the tibia and have pain with three-point stress. (Magee, p. 856) If a stress fracture is suspected, a tuning fork may be used at the suspected fracture site to provoke symptoms (Goodman, p. 1031).

A patient who is an avid runner had insidious onset of anterior leg pain that is localized, sharp, shooting in nature, and especially aggravated with heel strike (initial contact) during walking and running. The pain first occurred after an increase in running speed and hardness of the training surface 2 weeks ago. Which of the following tests and measures is MOST appropriate to determine the cause of the pain? 1. Compartmental pressure testing 2. Joint range of motion and strength 3. Biomechanical gait assessment 4. Vibration and percussion testing

1. Reduce the insulin infusion dose. 1. One benefit of using an insulin pump is being able to change the insulin delivery. Reducing or suspending the insulin will safely address the decreased blood glucose level (which is less than 100 mg/dL [5.5 mmol/L]) without the need for a carbohydrate snack (pp. 528, 618). 2. Increasing the insulin infusion dose would further decrease the blood glucose level. The patient's blood glucose is already less than normal. (p. 507) 3. Insulin pump delivery should be reduced, not maintained, in a patient whose current blood glucose level is 95 mg/dL (5.3 mmol/L) (p. 520). If the patient continued at the current rate, the insulin level would be too low (p. 507). 4. Rest would not address the problem, given the blood glucose level. Blood glucose less than 100 mg/dL (5.5 mmol/L) is abnormal and could be addressed by adjusting the insulin infusion dose. (p. 507)

A patient who uses an insulin pump is beginning an outpatient aerobic conditioning session. The patient's current blood glucose level is 95 mg/dL (5.3 mmol/L). Which of the following responses is MOST appropriate for the patient? 1. Reduce the insulin infusion dose. 2. Increase the insulin infusion dose. 3. Continue insulin infusion at the current metered dose. 4. Rest for 1 hour, then remeasure blood glucose level

4. Loss of pain and temperature sensation in the right side of the body 1. Corticospinal fibers that supply motor activity are located in the anteromedial aspect of the entire brainstem. Since the infarction is located laterally, it would not include any corticospinal fibers supplying the lower motor neurons. 2. Corticospinal fibers that supply motor activity are located in the anteromedial aspect of the entire brainstem. Since the infarction is located laterally, it would not include any corticospinal fibers supplying the lower motor neurons. 3. Discriminative touch and proprioceptive information travel through the brainstem in the medial lemniscus, which lies medially. Because it lies medially, it would not be involved in lateral medullary syndrome. 4. The spinothalamic tract in the brainstem, which carries sensory information regarding pain and temperature, lies laterally and thus would be impaired in a lateral medullary stroke. The spinothalamic fibers cross at the spinal cord level, and if they are impaired in the medulla, a deficit would manifest on the opposite side of the body. This would also be combined with cranial nerve signs on the same side of the lesion. A combination of cranial nerve signs on one side with other deficits (sensory or motor) in the body is a clear sign of brainstem dysfunction.

A patient with a brainstem infarction resulting in left lateral medullary syndrome is MOST likely to demonstrate which of the following symptoms? 1. Hemiparesis in the left side of the body 2. Hemiparesis in the right side of the body 3. Loss of discriminative touch and proprioception in the left side of the body 4. Loss of pain and temperature sensation in the right side of the body

2. cogwheel resistance to passive range of motion and tremor in the right extremities. 1. The clasp-knife phenomenon occurs as a result of injury to descending motor pathways from the cortex or brainstem, not the basal ganglia. 2. Cogwheel resistance to passive manipulation results from lesions of the basal ganglia. Tremor is also seen with injury to the basal ganglia. 3. The clasp-knife phenomenon occurs as a result of injury to descending motor pathways from the cortex or brainstem, not the basal ganglia. 4. Rigidity rather than hypotonia is seen with lesions of the basal ganglia

A patient with a focal lesion in the region of the left basal ganglia will MOST likely demonstrate: 1. clasp knife resistance to active range of motion and hypotonia in the left extremities. 2. cogwheel resistance to passive range of motion and tremor in the right extremities. 3. clasp knife resistance to passive range of motion and tremor in the right extremities. 4. cogwheel resistance to active range of motion and hypotonia in the left extremities

1. lower extremity strength. 1. Muscle weakness affects the performance of functional movement. Weakness in the extremities interferes with functional use either in weight-bearing (in this case, sitting to standing) or during movement in space. (pp. 725, 731) 2. Impairment of static balance is assessed in quiet standing position with the therapist applying perturbation. The goal is for the patient to remain still. This is not likely to be an issue if the patient can stand and walk independently. (p. 660) 3. Although impairment of the somatosensory system can be a cause of balance impairment, the independence of transfers and gait indicate that this is not a significant impairment for this patient (p. 682). 4. Dynamic balance integrity is evidenced by a patient who can recover balance with voluntary weight-shifting. The ability to stand and walk indicates that the patient had adequate dynamic balance. (p. 660)

A patient with a mild hemiparesis is able to stand and walk independently with a quad cane, but reports difficulty transferring from sitting to standing. A physical therapist is MOST likely to find the cause of the difficulty by testing the patient's: 1. lower extremity strength. 2. static standing balance. 3. joint position sense. 4. dynamic standing balance

4. Urethra 1. The ureters connect the kidneys to the bladder and are not in proximity to the prostate gland. 2. During the pelvic part of its course, the ductus deferens maintains direct contact with the peritoneum; no other structure intervenes between them. A vasectomy is the procedure in which the ductus deferens is ligated. It is not affected by prostatectomy. 3. The levator ani muscle is a skeletal muscle that attaches to the pelvis and is not DIRECTLY impacted by removal of the prostate. When the prostate is removed, there will be an expected period of incontinence lasting up to 6 weeks. Strengthening of the pelvic floor muscles can be a compensatory strategy for the loss of the support previously supplied by the prostate. 4. Prostatectomy indicates removal of the prostate gland. The prostate gland provides mechanical support to the urethra between the bladder neck and the penis.

A patient with a recent prostatectomy has urinary incontinence. Loss of support to which of the following structures is MOST likely contributing to the incontinence? 1. Ureter 2. Ductus deferens 3. Levator ani 4. Urethra

1. A neuroma 1. A neuroma is a collection of axons and fibrous tissue that can cause sharp, shooting, and localized pain (Lusardi, p. 707). Localized hypersensitivity may be an indicator that a neuroma has developed. 2. Inadequate prosthetic tibial relief may result in skin breakdown, which would be seen and is not indicated in the stem (Lusardi, p. 714). 3. Adherent scar tissue near the end of the bone is a particular problem that may lead to skin breakdown, which is not indicated in the stem (Lusardi, p. 819). 4. Clinical manifestation of pain with osteomyelitis may be described as deep, constant, and increasing with weight-bearing when present in the lower extremity. Patients may report local pain and swelling, which is not indicated in the stem. (Goodman, p. 1236

A patient with a transtibial amputation is being treated by a physical therapist for gait training with a prosthesis. The patient reports tingling and shooting pain at the end of the residual limb. The pain occurs whether or not the patient is wearing the prosthesis. The pain is MOST likely caused by which of the following? 1. A neuroma 2. Inadequate prosthetic tibial relief 3. Distal soft-tissue adhesions 4. Osteomyelitis

3. Decreased inspiratory reserve volume 1. Increased residual volume is seen in obstructive lung disease and not in restrictive lung disease (Frownfelter, pp. 139-141). This would not be seen with ascites and would not be present in the patient described. 2. Increased functional residual capacity is seen in obstructive lung disease and not in restrictive lung disease (Frownfelter, pp. 139-141). This would not be seen with ascites and would not be present in the patient described. 3. Ascites is a condition where there is increased fluid in the peritoneal cavity, causing distention (Moore, p. 233). This distention puts increased pressure upon the diaphragm and thoracic cavity, which may lead to a restrictive lung pattern. Decreased inspiratory reserve volume is a component of restrictive lung disease (Frownfelter, pp. 139-141). 4. Decreased forced expiratory volume in 1 second (FEV1) is commonly seen in obstructive lung disease, caused by either secretions or bronchospasm (Frownfelter, pp. 139-141). These do not result from ascites and would not be present in the patient described

A patient with advanced liver disease has developed severe ascites. This condition will MOST likely have which of the following effects on the patient's respiratory system? 1. Increased residual volume 2. Increased functional residual capacity 3. Decreased inspiratory reserve volume 4. Decreased forced expiratory volume in 1 second (FEV1)

3. UTI 1. Pulmonary dysfunction is a long-standing issue in patients who have C6 quadriplegia and would not cause the recent change in status (O'Sullivan, p. 898). 2. Autonomic dysreflexia results in hypertension, not left shoulder pain or fever, and has a rapid onset (O'Sullivan, p. 896). 3. Urinary tract infection is correct because spasticity can be influenced by internal factors such as a urinary tract infection. Also, a urinary tract infection can be associated with fever and can refer pain to the ipsilateral shoulder. (Goodman, p. 388) 4. Symptoms of deep vein thrombosis include swelling, erythema, and heat in a localized area (O'Sullivan, p. 901). These symptoms are not described in the stem

A patient with complete C6 tetraplegia reports a 3-day history of increased lower extremity spasticity, left shoulder pain, and fever. Which of the following conditions is MOST likely responsible for these findings? 1. Decreased respiratory function 2. Autonomic dysreflexia 3. Urinary tract infection 4. Deep vein thrombosis

1. Left sidelying with right hip abduction, extension, and lateral (external) rotation 1. The photograph shows a positive Trendelenburg sign, which occurs when the patient stands on one foot and the pelvis drops on the opposite side, indicating weakness of the stance leg's hip abductors (Magee, pp. 675, 1009). To strengthen the right hip abductors, the patient should lie on the left side to work the right lower extremity against gravity (Kisner, pp. 751-752). 2. The photograph indicates right hip abductor weakness (Magee, pp. 675, 1009). Right sidelying with left hip abduction, extension, and lateral (external) rotation would work the left hip abductors (Kisner, pp. 751-752). 3. The photograph indicates right hip abductor weakness (Magee, pp. 675, 1009). Supine hip flexion with the knee in extension will strengthen the hip flexors and quadriceps muscles (Kisner, p. 832). 4. The photograph indicates right hip abductor weakness (Magee, pp. 675, 1009). Right sidelying with right hip adduction and flexion will work the hip adductors (Kisner, p. 754).

A patient with hip pain demonstrates a trendelenburg gait pattern shown in the photograph. Which of the following exercises would be MOST beneficial to reduce this deviation? 1. Left sidelying with right hip abduction, extension, and lateral (external) rotation 2. Right sidelying with left hip abduction, extension, and lateral (external) rotation 3. Supine hip flexion with the knee in extension 4. Right sidelying with right hip adduction and flexion

1. Deep breathing exercises 1. In the acute care setting, deep breathing and coughing should be encouraged (p. 1698). 2. Patients with myasthenia gravis have intact sensation and do not typically have problems with skin breakdown. The cardinal features are muscle weakness with fatigue. (p. 1697) 3. Because the fundamental defect in myasthenia gravis is failure of neural transmission at the neuromuscular junction, electrical stimulation would not be appropriate (p. 1696). 4. A patient who has myasthenia gravis should avoid strenuous exercise. Frequent rest periods help conserve energy and give muscles a chance to regain strength. (p. 1698)

A patient with myasthenia gravis has been transferred from intensive care to an acute care neurology unit after a myasthenic crisis. Which of the following physical therapy interventions is MOST appropriate for the patient? 1. Deep breathing exercises 2. Instruction in skin inspection 3. Neuromuscular electrical stimulation 4. Treadmill training with progressive incline

4. Hypermobility of the right temporomandibular joint 1. The jaw jerk reflex is used to test the trigeminal nerve (CN V), but the question does not indicate neurological deficits (p. 1366). 2. The left temporomandibular joint is not hypermobile, so an indentation would not be expected (p. 1361). 3. Facial nerve paresthesias are not expected with temporomandibular joint hypermobility. The temporomandibular joint is primarily supplied by three nerves that are part of the mandibular division of the trigeminal nerve (CN V). (p. 1347) 4. A deviation during opening is associated with hypomobility toward the temporomandibular joint deviation and hypermobility contralaterally (p. 1361).

A patient with pain on the right side of the face also has tenderness at the right temporomandibular joint. A physical therapist notes deviation of the mandible toward the left at the end of the available range of mouth opening. Which of the following additional findings would be MOST expected? 1. Abnormal jaw reflex 2. Indentation behind the left condyle of the mandible 3. Paresthesia in the facial nerve (CN VII) distribution on the right 4. Hypermobility of the right temporomandibular joint

Neurapraxia. 1. Neurapraxia involves blockage that stops or slows conduction across that point in the nerve. Conduction above and below the blockage is usually normal. Recovery is possible. 2. With axonotmesis, the neural tube is intact, but axonal damage has occurred with Wallerian degeneration. Surgical intervention may be required. 3. Neurotmesis involves total loss of axonal function, with disruption of the neural tube. Surgical intervention is usually required. 4. The nerve is completely separated. Surgical intervention is required

A patient with which of the following injuries has the BEST prognosis for nonsurgical recovery? 1. Neurapraxia 2. Axonotmesis 3. Neurotmesis 4. Complete nerve severance

1. P 1. Junctional rhythm originates from the atrioventricular junction instead of the sinoatrial node, which normally causes the P wave. Therefore, the P wave will be missing. R, S, T waves come from the ventricles after stimulation from the atrioventricular junction and will be unaffected. 2. Junctional rhythm originates from the atrioventricular junction instead of the sinoatrial node, which normally causes the P wave. Therefore, the P wave will be missing. R waves come from the ventricles after stimulation from the atrioventricular junction and will be unaffected. 3. Junctional rhythm originates from the atrioventricular junction instead of the sinoatrial node, which normally causes the P wave. Therefore, the P wave will be missing. S waves come from the ventricles after stimulation from the atrioventricular junction and will be unaffected. 4. Junctional rhythm originates from the atrioventricular junction instead of the sinoatrial node, which normally causes the P wave. Therefore, the P wave will be missing. T waves come from the ventricles after stimulation from the atrioventricular junction and will be unaffected.

A patient's electrocardiogram shows a junctional rhythm. The patient's heart rate is 60 bpm and regular. Which of the following waves will MOST likely be absent from the rhythm strip? 1. P 2. R 3. S 4. T

4. substitute for the lack of muscle activity. 1. A person who has spina bifida uses a knee-ankle-foot orthosis when motor function is weak or absent or to address knee instability. It is not commonly used for incoordination of muscles. 2. A knee-ankle-foot orthosis is not able to facilitate muscle activity. 3. A knee-ankle-foot orthosis is not commonly used to prevent muscle contractures. It may be used when contractures are already present, if these contractures prevent upright positioning. 4. A knee-ankle-foot orthosis is commonly used when muscles of the knee are weak and muscles of the ankle are absent. It substitutes for lack of muscle activity

A person with spina bifida uses a knee-ankle-foot orthosis to: 1. provide support for muscle incoordination. 2. facilitate muscular activity. 3. prevent development of muscle contractures. 4. substitute for the lack of muscle activity.

1. Swallowing difficulties 1. Patients with an initial onset of bulbar and respiratory weakness tend to have a more rapid progression to death than patients whose weakness begins in the distal extremities. 2. Cognitive deficits are not associated with amyotrophic lateral sclerosis. 3. Sphincter control problems are not a component of amyotrophic lateral sclerosis. 4. Musculoskeletal pain is not predictive for prognosis in amyotrophic lateral sclerosis

A physical therapist is obtaining the medical history of a patient with amyotrophic lateral sclerosis. Which of the following is MOST important to ask about in order to determine the prognosis for this patient? 1. Swallowing difficulties 2. Cognitive deficits 3. Bowel and bladder function 4. Neck pain

2. Decreased heart rate response to exercise 1. Early mobilization with cardiopulmonary conditioning will improve vascular reflexes and the responsiveness of blood vessels in the lower extremity to constrict, reducing postural hypotension and dizziness. This would be demonstrated with an increased cardiovascular response upon rising from the supine position. (Reid, p. 290) 2. Exercise will increase the patient's stroke volume secondary to increased myocardial contractility. The increased stroke volume will result in a reduced heart rate response with exercise. The intensity of the exercises is directly correlated to the heart rate response. (Kisner, p. 252) 3. Early mobilization increases minute ventilation (tidal volume and respiratory rate) and does not decrease heart rate response (Reid, p. 290). 4. There is a decrease in cardiovascular resistance with training. Vascular reflexes and the responsiveness of blood vessels improve. (Reid, p. 290; Kisner, p. 245)

A physical therapist is providing supervised exercise to a patient who has been restricted to extended bed rest. After 2 weeks of intervention, which of the following measures would BEST reflect cardiopulmonary system improvement? 1. Delayed cardiovascular response upon rising from supine position 2. Decreased heart rate response to exercise 3. Decreased respiratory rate in response to exercise 4. Increased cardiovascular peripheral resistance

1. Repeat the motion with the palm upward. 1. Extending the shoulder with lateral (external) rotation allows the long head of the triceps to substitute for the latissimus dorsi. The therapist must ensure that the patient understands the desired motion before determining whether to add resistance or change the patient's position. (pp. 58-60) 2. Sitting is not a position used to test latissimus dorsi strength. Sitting would decrease the effect of gravity. (pp. 57-58) 3. The patient must be able to perform the proper motion against gravity before resistance can be added (p. 59). 4. The therapist may have chosen to start testing in the gravity-eliminated position (sidelying) if less than Fair (3/5) strength was suspected, but, having chosen to start in the against-gravity position, the therapist should first determine whether the patient truly cannot perform the motion against gravity before changing position (p. 60)

A physical therapist is testing the strength of a patient's latissimus dorsi with the patient in prone position and arms at the side with palms facing the ceiling. Following instruction in the desired motion, the patient lifts the arm off the table after first turning the palm downward. The therapist should ask the patient to do which of the following actions NEXT? 1. Repeat the motion with the palm upward. 2. Extend the arm while seated without back support. 3. Repeat the motion while the therapist adds resistance. 4. Extend the arm while in sidelying position with the upper arm supported

2. prolonged stretch technique. 1. Manual resistance-through-range technique is appropriate for a patient with a primary problem of muscular weakness, which is not the case in this patient example (pp. 178-179). 2. Prolonged stretch technique has ample evidence to suggest that it is most efficient for reducing this patient's primary problem of muscle stiffness (pp. 86, 87). 3. Contract-relax technique requires that a patient exert voluntary control of muscle actions, which is not likely given this patient's severe cognitive impairments (p. 94). 4. Neurodevelopmental techniques, such as rhythmic stabilization and alternating isometrics, are designed to improve postural stability or dynamic stability of a joint. Although this may assist the patient's performance, it does not address the primary problem of muscle stiffness. (pp. 214-215)

A physical therapist is treating a patient with a traumatic brain injury who has severe cognitive impairments and bilateral hip and knee flexion contractures. The MOST appropriate intervention to treat the patient's contractures is: 1. manual resistance-through-range technique. 2. prolonged stretch technique. 3. contract-relax technique. 4. neurodevelopmental techniques.

3. Report concerns about the patient's treatment adherence to the referring physician. 1. The patient could be instructed in pain relief strategies, because the family does not have the child's best interest in mind, but the referring physician should be contacted first in this case. 2. Discussing the patient's case with the baseball coach would violate the privacy rules of the Health Insurance Portability and Accountability Act (HIPAA). 3. Concerns about the patient's recovery should be communicated to the referring practitioner prior to action. 4. Taping is used to prevent further injury during the activity. Taping is not indicated to allow participation when rest is recommended

A physical therapist is treating an adolescent patient who has medial elbow pain. The patient has been advised by the physician to rest the elbow, but the patient's parents insist that the patient play in an upcoming baseball tournament. Which of the following actions is MOST appropriate for the therapist? 1. Instruct the family in pain relief strategies. 2. Discuss concerns about the patient's condition with the patient's baseball coach. 3. Report concerns about the patient's treatment adherence to the referring physician. 4. Tape the patient's elbow.

2. Ankylosing spondylitis 1. Osteoarthritis is a degenerative disease that mainly affects the hands, hips, and knees, causing disability. Manifestations of the disease do not compromise ventilation, and, therefore, use of an incentive spirometer would not be beneficial. (Goodman, p. 1304) 2. Ankylosing spondylitis is a chronic inflammatory disease that leads to spinal and chest wall rigidity (Hillegass, p. 246; Frownfelter, p. 538). Due to the chest wall restriction, there is a decrease in total lung capacity, vital capacity, and inspiratory muscle function (Frownfelter, p. 538). The device in the photograph is an incentive spirometer, which encourages deep breathing and extended inspiration, leading to an increase in vital capacity and chest wall mobility (Frownfelter, pp. 695-696). This device would be beneficial for a patient who has ankylosing spondylitis. 3. Reiter syndrome is a form of reactive arthritis that can follow an infection by a microbial pathogen. Symptoms include urethritis, conjunctivitis, and arthritis. There is no compromise to ventilation indicated, and, therefore, use of an incentive spirometer would not be beneficial. (Goodman, pp. 1343-1344) 4. Psoriatic arthritis occurs in 20% of patients who have psoriasis and leads to joint disorders due to the development of inflammatory synovitis. The disease is most likely to affect the distal interphalangeal joints of the hands. Symptoms do not compromise ventilation; therefore, use of an incentive spirometer would not be beneficial. (Goodman, pp. 1341-1342)

A spirometer would be MOST beneficial for a patient who has which of the following conditions? 1. Osteoarthritis 2. Ankylosing spondylitis 3. Reiter syndrome 4. Psoriatic arthritis

1. continue with treatment. Normal change

A therapist is treating a patient who recently had a myocardial infarction. At the beginning of treatment, blood pressure was 120/80 mm Hg and heart rate was 90 beats/min. Midway through treatment, blood pressure was 130/84 mm Hg and heart rate was 105 beats/min. The BEST action for the therapist to take is to: 1. continue with treatment. 2. increase the intensity of treatment. 3. stop the treatment, and notify the physician. 4. decrease the intensity of the next treatment.

2. improved neuromuscular recruitment. Author: Kisner C, Colby LA Title: Therapeutic Exercise: Foundations and Techniques Edition: 6 Publisher: F.A. Davis Year: 2012 Pages: 168-169 1. Although transformation of type IIB to type IIA fibers occurs in the early weeks of resistance training, transformation from slow-twitch to fast-twitch fibers is unlikely (p. 169). 2. The initial rapid gain in the tension-generating capacity of skeletal muscle is largely attributable to neural responses, including increased recruitment in number of motor units firing and increased rate of synchronization of firing (p. 168). 3. Muscle-fiber hyperplasia is an increase in the number of muscle fibers, and if it occurs, it is in response to heavy resistance training and only accounts for a small percentage of the increase in strength (p. 168). 4. Muscle hypertrophy, or increase in the size of individual muscle fibers, requires an extended period (4-8 weeks) of moderate-intensity to high-intensity resistance training (p. 168). One week is too short of a duration for such change

After 1 week of a progressive resistance exercise training program, an individual demonstrates significant strength gains. The MOST likely explanation for the observed strength gains is: 1. an increased ratio of fast- to slow-twitch fibers. 2. improved neuromuscular recruitment. 3. muscle-fiber hyperplasia. 4. muscle hypertrophy

2. Upper trunk of the brachial plexus 1. An axillary nerve lesion will not cause weakness in elbow flexion. 2. The symptoms are consistent with damage to the C5 and C6 nerve roots. 3. Symptoms of a long thoracic nerve lesion would be more widely distributed and distal. 4. A lesion of the musculocutaneous nerve would not cause lateral shoulder pain.

After a 30-day backpacking trip, a patient has pain over the lateral aspect of the shoulder. Upon examination of the patient, the physical therapist notes weakness of elbow flexion and shoulder abduction on the same side as the pain. A lesion at which of the following sites is MOST likely the source of these symptoms? 1. Axillary nerve 2. Upper trunk of the brachial plexus 3. Long thoracic nerve 4. Musculocutaneous nerve

3x/week 1. Once a week is not a sufficient exercise volume to promote tissue adaptation. 2. Evidence suggests that resistance training programs can be performed safely and appropriately both by children and adolescents. Yet to avoid injury, it is important to allow an adequate recovery (2 to 3 days) between training sessions. 3. Five times/week does not allow sufficient recovery time between sessions and could increase risk for overuse injury. 4. Seven times/week will not allow sufficient recovery time between sessions and could increase risk for overuse injury

An 11-year-old patient who is recovering from a knee injury is beginning a resistance exercise program to strengthen the quadriceps. Which of the following exercise schedules is MOST appropriate for the patient? 1. 1 time/week 2. 3 times/week 3. 5 times/week 4. 7 times/week

4. Provide postural correction at the pelvis. 4. This technique will address the posterior pelvic tilt associated with "C"-shaped posture.

An 18-month-old child with developmental delay collapses into a rounded "C"-shaped posture of forward flexion when placed in a sitting position. Which of the following techniques is most appropriate INITIALLY to improve the child's posture? 1. Seat the child in a chair with an abduction wedge. 2. Apply approximation through the spinal joints. 3. Present toys from above so that the child must look up. 4. Provide postural correction at the pelvis.

1. Massed Author: Shumway-Cook A, Woollacott MH Title: Motor Control: Translating Research into Clinical Practice Edition: 5 Publisher: Wolters Kluwer Year: 2017 Pages: 34-351. Massed practice refers to a sequence of practice and rest times in which rest time is much less than the practice time (p. 34). 2. Distributed practice refers to practice intervals in which the practice time is equal to or less than the rest time (p. 34). 3. Blocked practice refers to a practice sequence organized around one task performed repeatedly, uninterrupted by practice of any other tasks. Therefore, this type of practice is not related to the amount of time spent on performing a task. (p. 35) 4. Random practice refers to a practice sequence in which several various tasks are ordered randomly across trials (p. 35)

An exercise session that includes 25 minutes of continuous practice and 5 minutes of rest BEST represents which of the following types of practice? 1. Massed 2. Distributed 3. Blocked 4. Random

2. The physical therapy evaluation will include tests that will specifically assess for nonorganic signs and behavioral symptoms. 1. The therapist can and should act as part of the medical team, which includes determining sincerity of patients to the extent possible and documenting a finding of malingering to assist in appropriate care as needed. To ignore this responsibility would be unfair to the insurance company, the malingering patient (e.g., in case a psychiatric referral is needed), and other patients who could have used the treatment resources used by the malingering patient. 2. All patients should be given the benefit of the doubt until the clinician, with a high degree of confidence, can rule out an organic cause for the pain. A number of tests can be performed to help determine if the symptoms are of a nonorganic nature, but it is very difficult to judge the intent (psychogenic versus intentional exaggeration for gain). 3. The therapist should not initiate patient care with preconceived ideas that are potentially harmful to the patient's success. Expressing such assumptions to the patient will bias the feedback the patient provides because the patient will be defensive, further skewing the results of the evaluative process. 4. As a member of a health care team, a physical therapist can play a role in documentation of malingering, once behavior that is characteristic of malingering is observed. The therapist should not recommend referral to a psychologist prior to performing the initial evaluation

During a phone call with a physical therapist regarding a patient referral, the physician who has referred the patient expresses a belief that the patient is fabricating symptoms for secondary gain and asks the therapist to provide documentation to demonstrate this impression. Which of the following responses is BEST for the therapist to provide in this situation? 1. Deciding whether a patient is fabricating symptoms for secondary gain is beyond the scope of physical therapy practice. 2. The physical therapy evaluation will include tests that will specifically assess for nonorganic signs and behavioral symptoms. 3. The patient will be confronted on issues of fabricating symptoms for secondary gain in an attempt to alter the behavior. 4. It is recommended that the patient be referred to a psychologist prior to initiating physical therapy.

1. left rectus femoris. 1. The described position posteriorly tilts the pelvis and, in the presence of left rectus femoris tightness, results in decreased left knee flexion. 2. The right biceps femoris would not be affected, because right knee extension is not assessed in the position. 3. The left iliopsoas would not cause decreased left knee flexion, because it is a hip flexor muscle. 4. The right iliotibial band is not being stretched in this position, and it would not contribute to decreased knee flexion

During flexibility testing, a patient is supine with knees flexed over the end of the table. With passive flexion of the right hip, the physical therapist observes that the patient's left knee extends. This is MOST likely due to tightness of the: 1. left rectus femoris. 2. right biceps femoris. 3. left iliopsoas. 4. right iliotibial band

2. Painful left knee 1. Right iliopsoas contracture may cause a shorter step length with the left lower extremity but not with the right lower extremity (Mansfield, p. 356). 2. Left knee pain will cause the patient to spend less time in left-sided stance, because the patient will try to minimize the time spent in stance (weight-bearing on the knee) to minimize the pain. Therefore, the patient will take a shorter step with the right lower extremity. (Magee, p. 1007). 3. Decreased ankle pronation would not have an effect on right-sided step length (Magee, p. 1005). 4. Gluteus medius weakness would be seen as an increase in lateral pelvic tilt, not step length (Mansfield, p. 357).

During gait evaluation, a physical therapist notes that a patient demonstrates a shorter step length with the right lower extremity. Which of the following problems is MOST likely the cause of the gait dysfunction? 1. Right iliopsoas contracture 2. Painful left knee 3. Decreased ankle pronation on the right 4. Left gluteus medius weakness

2. Vertebral rotation 1. The forward bend test will show a rotary component of the spine if there is a scoliosis. Excessive thoracic kyphosis is not determined by the forward bend evaluation. Excessive kyphosis should be viewed from the side. (p. 205) 2. Scoliosis during forward bending is indicated by a spinal curve. This is called a rib hump and is caused by rotation of the vertebral bodies. (pp. 205, 240) 3. The purpose of the forward bend test is to assess for the rotary component of the scoliosis, not lateral spinal deviation (p. 240). 4. The purpose of the forward bend test is to assess for the rotary component of the scoliosis, not lumbar lordosis (p. 240).

During scoliosis screening, the forward bend evaluation is conducted to determine the presence of which of the following clinical features? 1. Excessive thoracic kyphosis 2. Vertebral rotation 3. Lateral spinal deviation 4. Lumbar lordosis

3. Unilateral vestibular lesion 1. Patients with benign paroxysmal positional vertigo most likely will not have a positive finding on the Romberg test. 2. Patients with cerebellar lesions are more likely to exhibit ataxic gait. 3. Patients with a unilateral vestibular lesion will experience vertigo, postural instability, oscillopsia, and disequilibrium. The wide-based gait is an attempt to minimize trunk rotation and movement of the head, which can increase sensory conflict and vertigo in the patient with a unilateral vestibular lesion. 4. Mononeuropathy of the sural nerve is unlikely to cause a balance deficit

Examination of a patient with balance dysfunction reveals the following: Romberg test: positive Gait: wide-based, slow, with decreased trunk rotation Loss of balance when asked to turn head while walking No sign of ataxia Based on these findings, which of the following diagnoses is MOST likely correct? 1. Benign paroxysmal positional vertigo 2. Cerebellar lesion 3. Unilateral vestibular lesion 4. Mononeuropathy of the sural nerve

4. Lateral weight-shifts in stance 1. Walking backward on an inclined surface does not address gluteus medius weakness and may be too difficult for this patient (p. 757). 2. Sidelying right hip abduction does not provide balance practice/training (p. 752). 3. Partial squats against a wall would not provide balance practice/training and will strengthen hip and knee extensors more than hip abductors (p. 756). 4. Lateral weight-shifting in stance can help to improve balance abilities and also involves gluteus medius activation. Any shift in body mass in a lateral direction involves hip abductors/adductors. (p. 264)

Following a left cerebrovascular accident, a patient displays mild balance deficits along with weakness in the gluteus medius. Which of the following activities is MOST appropriate for this problem? 1. Walking backward on an inclined surface 2. Sidelying right hip abduction 3. Partial squats against a wall 4. Lateral weight-shifts in stance

4. WNL 1. A sensory nerve conduction test only measures the distal component of a peripheral nerve. The technique would not be able to detect abnormalities in a lesion proximal to the dorsal root ganglion. If only sensory roots are injured, no electromyogram changes occur. 2. A sensory nerve conduction test only measures the distal component of a peripheral nerve. The technique would not be able to detect abnormalities in a lesion proximal to the dorsal root ganglion. If only sensory roots are injured, no electromyogram changes occur. 3. A sensory nerve conduction test only measures the distal component of a peripheral nerve. The technique would not be able to detect abnormalities in a lesion proximal to the dorsal root ganglion. If only sensory roots are injured, no electromyogram changes occur. 4. A sensory nerve conduction test only measures the distal component of a peripheral nerve. The technique would not be able to detect abnormalities in a lesion proximal to the dorsal root ganglion. If only sensory roots are injured, no electromyogram changes occur

For a patient with a lesion proximal to the dorsal root ganglion, a sensory nerve conduction velocity test will reveal that conduction times are: 1. decreased in amplitude. 2. markedly decreased. 3. markedly increased. 4. within normal limits

1. Independence in bowel care 1. Among the variables that significantly increase life satisfaction in individuals with traumatic brain injury is bowel independence (p. 760). 2. Among the variables that significantly increase life satisfaction in individuals with traumatic brain injury is being married (p. 760). 3. Younger age (less than 20 years) is associated with better outcome but not with a higher level of life satisfaction (pp. 759-760). 4. Rate of recovery is not associated with life satisfaction (pp. 759-760)

In determining the prognosis of an individual with a traumatic brain injury, which of the following variables is MOST associated with higher level of life satisfaction? 1. Independence in bowel care 2. Being unmarried 3. Age less than 20 years 4. Rapid initial recovery from injury

2. Tibial

On initial evaluation of a patient, a physical therapist finds a weakness in plantar flexion, forefoot inversion, and toe flexion. The therapist should suspect involvement of which of the following peripheral nerves? 1. Medial plantar 2. Tibial 3. Common peroneal 4. Femoral

3. Overhead activities 1. Pain and weakness with the pictured test (empty can test) indicate partial rotator cuff tear or impingement. Rotator cuff strengthening is indicated. 2. Joint mobilization is not contraindicated for a patient who has a partial rotator cuff tear or impingement. 3. Overhead activities are painful and should be avoided in the early stages of treatment for a patient who has a partial rotator cuff tear or impingement. Work should occur only in the pain-free range. 4. Deep friction massage can be used for pain relief and to decrease scar tissue for a patient who has a partial rotator cuff tear or impingement.

Pain and weakness were elicited with the empty can test. Which of the following should be AVOIDED during the early treatment phase? 1. Rotator cuff strengthening 2. Joint mobilization 3. Overhead activities 4. Deep friction massage

3. alveolar dilation. 1. Patients with emphysema have normal or slightly elevated partial pressure of arterial carbon dioxide (PaCO2) (p. 88), not hypocapnia (or decreased CO2). 2. Patients who have emphysema tend to breathe with accessory muscles of respiration (p. 87), which may lead to hypertrophy, not atrophy of those muscles. 3. Emphysema is characterized by abnormal and permanent enlargement of the air spaces distal to the terminal nonrespiratory bronchioles accompanied by destructive changes of the alveolar walls (p. 86). 4. Emphysema is characterized by abnormal and permanent enlargement of the air spaces distal to the terminal nonrespiratory bronchioles accompanied by destructive changes of the alveolar walls (p. 86). There is no involvement of the phrenic nerve in this condition

Patients with advanced emphysema experience difficulty in breathing during exercise because of: 1. hypocapnia. 2. atrophy of secondary breathing muscles. 3. alveolar dilation. 4. damage to the phrenic nerve

2. Femoral length disparity 1. Knee and ankle landmarks are level; the tibia is not involved. 2. Because of level knee and ankle landmarks and normal pelvic symmetry, the leg length discrepancy is due to femoral length disparity. 3. Sacral nutation is not present in this scenario. 4. Hemivertebrae in the lumbar spine would not affect hip landmark height.

Postural screening of a patient reveals that the greater trochanters are not level in the standing position. The pelvis is not rotated or tilted, and the knee and ankle landmarks are level in the standing position. The findings indicate which of the following impairments? 1. Tibial length disparity 2. Femoral length disparity 3. Nutation of the sacroiliac joint 4. Hemivertebrae in the lumbar spine

1. less than or equal to 60% of maximum heart rate for 45 to 60 minutes, 5 to 7 days/week. 1. Initial exercise training for obese individuals should be moderate (40% to 60% of maximum heart rate) and eventually progress to higher intensities (50% to 75%). Frequency should be 5-7 days/week; duration should be 45-60 minutes/session. 2. Eighty percent to 90% of maximum heart rate is too high of an intensity; and 3 days a week is not a high enough frequency. 3. Fifteen to 20 minutes is not a long enough duration; and 3 days a week is not a high enough frequency. 4. Eighty percent to 90% of maximum heart rate is too high of an intensity; 15-20 minutes is not a long enough duration.

The BEST exercise level for an aerobic program for weight loss is: 1. less than or equal to 60% of maximum heart rate for 45 to 60 minutes, 5 to 7 days/week. 2. 80% to 90% of maximum heart rate for 45 to 60 minutes, 3 days/week. 3. less than or equal to 60% of maximum heart rate for 15 to 20 minutes, 3 days/week. 4. 80% to 90% of maximum heart rate for 15 to 20 minutes, 5 to 7 days/week

1. Anterior interosseous nerve 1. The image shows an anterior interosseous syndrome (Kiloh-Nevin syndrome) in the patient's left hand. The patient is unable to flex the distal phalanx of the thumb and index fingers (1st and 2nd digits) because the anterior interosseous nerve, which supplies the flexor pollicis longus and the radial half of the flexor digitorum profundus, is entrapped. (p. 411) 2. With entrapment of the radial nerve, all extensor muscles of the forearm would be affected (p. 416). The image shows a deficit in the pinch of the thumb and index finger (1st and 2nd digits). 3. Entrapment of the posterior interosseous nerve results in functional wrist drop (p. 416). The image shows a deficit in the pinch of the thumb and index finger (1st and 2nd digits). 4. When the ulnar nerve is affected, the patient cannot fully adduct the little finger (5th digit) and hold the finger abducted and extended (p. 415). The image shows a deficit in the pinch of the thumb and index finger (1st and 2nd digits)

The condition shown for the patient's left hand in the photograph is MOST likely caused by entrapment of which of the following nerves? 1. Anterior interosseous nerve 2. Radial nerve 3. Posterior interosseous nerve 4. Ulnar nerve

4. Flexing one hip with the same knee extended in supine 1. With a herniated disc, symptoms can increase with forward bending. The patient in the photograph is displaying a flexed lumbar spine. Reaching for the toes in long sitting position promotes trunk flexion and would increase the patient's symptoms. 2. With a herniated disc, symptoms can increase with forward bending. The patient in the photograph is displaying a flexed lumbar spine. Reaching for the toes in standing position promotes trunk flexion and would increase the patient's symptoms. 3. With a herniated disc, symptoms can increase with forward bending. The patient in the photograph is displaying a flexed lumbar spine. Pulling one knee to the opposite shoulder promotes trunk flexion and would increase the patient's symptoms. 4. In supine position, the lumbar spine would be in neutral, avoiding a flexed posture.

The patient shown in the photograph has a history of a herniated nucleus pulposus in the lumbar spine. Which of the following exercises is BEST for improving the patient's muscle length? 1. Reaching for toes in long sitting 2. Reaching for toes in standing, keeping the knees straight 3. Pulling one knee to the opposite shoulder in supine 4. Flexing one hip with the same knee extended in supine

3. Shoulder impingement syndrome 1. The photograph shows the Hawkins-Kennedy test. Anterior glenohumeral instability is not tested with the Hawkins-Kennedy test. The Hawkins-Kennedy test is used to test for impingement signs for the diagnosis of subacromial bursitis or rotator cuff pathology. (p. 630) 2. The photograph shows the Hawkins-Kennedy test. Cubital tunnel syndrome is not tested with the Hawkins-Kennedy test. The elbow flexion test or Tinel sign is used to test for cubital tunnel syndrome. (pp. 738-739) 3. The photograph shows the Hawkins-Kennedy test, which is used to test for impingement syndrome of the shoulder (p. 630). 4. The photograph shows the Hawkins-Kennedy test. Thoracic outlet syndrome is not tested with the Hawkins-Kennedy test. It is tested with the Roos test. (p. 1300)

The photograph shows the Hawkins-Kennedy test. Shoulder pain during the test shown in the photograph MOST likely indicates which of the following pathologies?1. Anterior glenohumeral instability 2. Cubital tunnel syndrome 3. Shoulder impingement syndrome 4. Thoracic outlet syndrome

1. Long thoracic 1. The long thoracic nerve innervates the serratus anterior. Weakness of the serratus anterior results in winging of the scapula, which is the pathological position shown in the photograph. (Magee, p. 281; Drake, pp. 726-727, 744) 2. The spinal accessory nerve innervates the sternocleidomastoid and trapezius. The trapezius adducts and upwardly rotates the scapula (Drake, pp. 895, 1024). The sternocleidomastoid flexes the head to the side and rotates the head to the contralateral side (Magee, p. 174). These muscles are not involved in the asymmetrical position shown in the photograph. 3. The axillary nerve innervates the deltoid and teres minor, which are not involved in the asymmetrical position shown in the photograph (Magee, pp. 177, 287). 4. The dorsal scapular nerve innervates the rhomboids, which elevate, retract, and downwardly rotate the scapula and are not involved in the asymmetrical position shown in the photograph (Drake, pp. 715-716, 744; Magee, p. 287)

The winging of the scapula is MOST likely due to a lesion in which of the following nerves? 1. Long thoracic 2. Spinal accessory 3. Axillary 4. Dorsal scapular

2. 0.5 milliamperes/cm2 2. To calculate current density, the current amplitude is divided by conductive surface area; therefore, 10/20 = 0.5.

What is the MAXIMUM current density that should be used to initiate iontophoresis when the current amplitude is 10 milliamperes and the conductive surface area is 20 cm2? 1. 0.2 milliamperes/cm2 2. 0.5 milliamperes/cm2 3. 5 milliamperes/cm2 4. 20 milliamperes/cm2

1. Facilitating cocontraction patterns and encouraging control in voluntary movement gradation 1. Athetoid cerebral palsy is characterized by involuntary movements that are slow and writhing. In therapy, the emphasis should be on facilitating cocontraction and encouraging control in voluntary movement. 2. Although strength training is indicated in children with cerebral palsy, a child with athetoid cerebral palsy lacks the control to consistently produce a maximal effort in a controlled movement. Therefore, the focus must first be on gaining control, then on traditional strength training. 3. A goal of therapy would not be to reinforce primitive reflexes. The primary need for this child is to gain controlled movement. 4. Children with athetoid cerebral palsy are less likely to have contractures, compared with children with other forms of cerebral palsy, because of the constant movement of limbs. This would not be the primary emphasis of therapy

Which of the following activities should be the PRIMARY emphasis of a physical therapy treatment program for a child who has athetoid cerebral palsy? 1. Facilitating cocontraction patterns and encouraging control in voluntary movement gradation 2. Increasing muscle strength using progressive resistive exercises 3. Facilitating use of primitive reflexes to perform gross motor tasks 4. Preventing development of contractures and ensuring full voluntary range of motion

3. Measure how far the patient can reach without losing balance. 1. The ability to stand with feet together gives information about static balance, not limits of stability. 2. Observing a patient's response to external perturbation does not test the limits of stability. 3. The limit of stability in any direction is how far outside the base of support a person can move without losing balance. Measuring how far the patient can reach is a test of limits of stability. 4. Postural sway in quiet standing position gives information about static balance, not limits of stability

Which of the following activities tests a patient's limits of stability in standing position? 1. Time how long the patient can stand with feet together. 2. Gently push on the patient's sternum and observe the response. 3. Measure how far the patient can reach without losing balance. 4. Observe postural sway with the patient standing in front of a grid.

2. Increase in interstitial tissue pressure and venous blood flow 1. The venous blood flow and lymphatic pressure would increase, not decrease, during the inflation (compression) phase. 2. During the compression phase, the arterial blood flow is reduced. Venous blood flow is enhanced, as is lymphatic flow. The interstitial pressure is increased, which forces fluid out of the interstitium into the lymphatic system. The compression of the veins and lymphatics forces fluids proximally. Arterial flow would increase during the deflation period. 3. Interstitial pressure is increased, not decreased, during the inflation (compression) phase. 4. Arterial flow will not increase during the compression phase.

Which of the following changes occur when using intermittent pneumatic compression during the inflation (compression) phase? 1. Decrease in venous blood flow and lymphatic pressure 2. Increase in interstitial tissue pressure and venous blood flow 3. Decrease in interstitial pressure and arterial blood flow 4. Increase in arterial and venous blood flow

1. Negative air pressure, with the direction of the airflow into the room from the adjacent space outside the room 1. In airborne infection isolation, the isolation area is under negative pressure, such that the direction of the airflow is from the outside adjacent space (e.g., the corridor) into the room.

Which of the following features are MOST characteristic of a room designed for airborne infection isolation? 1. Negative air pressure, with the direction of the airflow into the room from the adjacent space outside the room 2. Negative air pressure, with the direction of the airflow from the room into the adjacent space outside the room 3. Positive air pressure, with the direction of the airflow into the room from the adjacent space outside the room 4. Positive air pressure, with the direction of the airflow from the room into the adjacent space outside the room

1. Pain with valgus stress testing and minimal instability with the elbow in 60° of flexion

Which of the following findings are MOST consistent with the presence of a grade I sprain of the medial (ulnar) collateral ligament of the elbow? 1. Pain with valgus stress testing and minimal instability with the elbow in 60° of flexion 2. Pain with varus stress testing and moderate instability with the elbow in 60° of flexion 3. No pain with valgus stress testing and moderate instability with the elbow in 5° of flexion 4. No pain with varus stress testing and minimal instability with the elbow in 5° of flexion

1. Triglyceride level of 135 mg/dL (1.5 mmol/L) Author: Goodman CC, Snyder TEK Title: Differential Diagnosis for Physical Therapists: Screening for Referral Edition: 5 Publisher: Elsevier Saunders Year: 2013 Pages: 434-4351. Triglyceride level below 150 mg/dL is normal. 2. Blood pressure reading equal to or greater than 130/85 mm Hg is a risk factor for the development of metabolic syndrome. 3. Fasting glucose of 100 mg/dL (5.5 mmol/L) or more is a risk factor for the development of metabolic syndrome. 4. Waist measurement greater than 35 inches (89 cm) for women or 40 inches (102 cm) for men is a risk factor for the development of metabolic syndrome

Which of the following findings is CONSISTENT with low risk for development of metabolic syndrome? 1. Triglyceride level of 135 mg/dL (1.5 mmol/L) 2. Blood pressure reading of 135/85 mm Hg 3. Fasting blood glucose level of 126 mg/dL (7.0 mmol/L) 4. Waist measurement of 41 in (104.1 cm)

4. Peritoneal irritation 1. Rebound tenderness at the abdomen is not consistent with muscle guarding (pp. 340, 342). 2. Rebound tenderness is not consistent with a superficial reflex of the umbilicus when assessing for a neurologic impairment (pp. 201-203). 3. Rebound tenderness is not consistent with muscle soreness but rather is a classic sign of peritonitis (p. 341). 4. Pain on release of pressure confirms rebound tenderness, a reliable sign of peritoneal inflammation (p. 342).

Which of the following findings is MOST consistent with the presence of rebound tenderness noted during palpation of the abdomen of a patient who has low back pain? 1. Muscle guarding 2. Superficial reflex 3. Muscle soreness 4. Peritoneal irritation

1. Infraspinatus tightness 1. The photograph shows the patient performing medial (internal) rotation with the hand behind the back. Because the infraspinatus is a lateral (external) rotator, tightness will limit medial (internal) rotation (Norkin, pp. 86, 97). 2. The photograph shows the patient performing medial (internal) rotation with the hand behind the back. Based on the convex-concave rule, a tight inferior joint capsule would limit abduction (Kisner, p. 134). Since the limitation is in medial (internal) rotation and not abduction, this option would be incorrect. 3. The photograph shows the patient performing medial (internal) rotation with the hand behind the back. The posterior deltoid could contribute to the limited range since part of its function is lateral (external) rotation. However, it also functions during extension, so this position could shorten the muscles, making it less likely than the infraspinatus to contribute to the limitation (Norkin, p. 86; Lippert, p. 151). 4. This photograph shows the patient performing medial (internal) rotation with the hand behind the back. Based on the convex-concave rule of the shoulder, anterior capsule tightness would limit extension and lateral (external) rotation (Kisner, pp. 132, 135)

Which of the following impairments is MOST likely causing the limitation depicted in the patient's right upper extremity? 1. Infraspinatus tightness 2. Tight inferior glenohumeral joint capsule 3. Posterior deltoid tightness 4. Tight anterior glenohumeral joint capsule

4. Manual lymphatic drainage, compression therapy, and decongestive exercises

Which of the following interventions are components of complete decongestive therapy (CDT) for patients who have lymphedema? 1. Manual lymphatic drainage, iontophoresis, and aquatic exercises 2. Manual lymphatic drainage, ultrasound, and decongestive exercises 3. Manual lymphatic drainage, electrical stimulation, and aquatic exercises 4. Manual lymphatic drainage, compression therapy, and decongestive exercises

3. Walking on a treadmill 1. For individuals who have vertebral osteoporosis or previous history of vertebral fractures, activities such as golfing, bowling, biking, rowing, sit-ups, or other exercise with a major component of spinal flexion, side bending, or spinal rotation should be avoided (p. 1224). 2. Swimming/water aerobics is an excellent physical activity, but it is not beneficial to offset the complications of osteoporosis or build bone density (p. 1224). 3. Walking is associated with changes in bone remodeling and results in greater bone mass (p. 1222). 4. For individuals who have vertebral osteoporosis or previous history of vertebral fractures, activities such as golfing, bowling, biking, rowing, sit-ups, or other exercise with a major component of spinal flexion, side bending, or spinal rotation should be avoided (p. 1224)

Which of the following interventions is MOST appropriate to assist with maintaining bone density for a patient with a history of osteoporosis and previous vertebral fractures? 1. Use of a rowing machine 2. Water aerobics 3. Walking on a treadmill 4. Performing sit-ups on a mat

3. Methotrexate (Trexall) 1. The patient in the photograph has rheumatoid arthritis (Goodman, p. 1323). Furosemide is a diuretic used for the treatment of hypertension, not the treatment of rheumatoid arthritis (Ciccone, p. 320). 2. The patient in the photograph has rheumatoid arthritis (Goodman, p. 1323). Metformin is indicated for diabetes, not rheumatoid arthritis (Ciccone, p. 519). 3. The patient in the photograph has rheumatoid arthritis. Methotrexate is a disease-modifying antirheumatic drug used in the treatment of rheumatoid arthritis. (Goodman, p. 1323; Ciccone, p. 240) 4. The patient in the photograph has rheumatoid arthritis (Goodman, p. 1323). Atenolol is a beta-adrenergic blocking agent used in the treatment of hypertension or angina, not rheumatoid arthritis (Ciccone, p. 311)

Which of the following medications is MOST likely to be used to treat RA 1. Furosemide (Lasix) 2. Metformin (Glucophage) 3. Methotrexate (Trexall) 4. Atenolol (Tenormin)

4. Deceleration (terminal swing) 1. Deceleration, not heel strike (initial contact), is the phase of the gait cycle where the hamstrings are most active in eccentrically controlling hip flexion and knee extension. With running or sprinting, the amount of strain put on the hamstrings is much greater, and the eccentric force needed to control hip flexion and knee extension can cause the hamstrings to be injured. 2. Deceleration, not midswing, is the phase of the gait cycle where the hamstrings are most active in eccentrically controlling hip flexion and knee extension. With running or sprinting, the amount of strain put on the hamstrings is much greater, and the eccentric force needed to control hip flexion and knee extension can cause the hamstrings to be injured. 3. Deceleration, not foot flat (loading response), is the phase of the gait cycle where the hamstrings are most active in eccentrically controlling hip flexion and knee extension. With running or sprinting, the amount of strain put on the hamstrings is much greater, and the eccentric force needed to control the hip flexion and knee extension can cause the hamstrings to be injured. 4. Deceleration is the phase of the gait cycle where the hamstrings are most active in eccentrically controlling hip flexion and knee extension. With running or sprinting, the amount of strain put on the hamstrings is much greater, and the eccentric force needed to control hip flexion and knee extension can cause the hamstrings to be injured.

Which of the following phases of running is MOST likely to aggravate a hamstring injury? 1. Heel strike (initial contact) 2. Midswing 3. Foot flat (loading response) 4. Deceleration (terminal swing)

4. Place a nonsterile ruler close to the wound to obtain measurements and then discard the ruler after use. 4. The ruler should not make contact with the wound, and the ruler should be limited to a single use.

Which of the following procedures is MOST appropriate for measuring a wound that has well-defined margins? 1. Clean the skin around the wound, place a nonsterile ruler on the wound to obtain measurements, and then clean the ruler for future use. 2. Clean the skin around the wound, place a nonsterile ruler on the wound to obtain measurements, and then discard the ruler after use. 3. Place a nonsterile ruler close to the wound to obtain measurements and then clean the ruler for future use. 4. Place a nonsterile ruler close to the wound to obtain measurements and then discard the ruler after use

1. Marching in place 1. Equilibrium is a dynamic reaction essential for upright posture and smooth transitional movements. Equilibrium is assessed by testing the body's ability to respond to a change in body position or surface support to maintain body alignment. Of the options, only marching in place requires a displacement of the center of gravity and a places a demand on the body to maintain upright posture. (Umphred, p. 393) 2. The rebound test is a nonequilibrium coordination test, involving contraction of the biceps, sudden release of resistance, and sudden contraction of the triceps to retrain limb movement (O'Sullivan, p. 219). 3. Heel on shin is a nonequilibrium coordination test (O'Sullivan, p. 219). 4. Finger to therapist's finger is a nonequilibrium coordination test (O'Sullivan, p. 218)

Which of the following procedures should be used to assess a patient's equilibrium? 1. Marching in place 2. Rebound test 3. Heel on shin 4. Finger to therapist's finger

3. Pain and paresthesia followed by a unilateral rash along the affected dermatome 3. Herpes zoster (shingles) is initially manifested as pain and paresthesia along the affected dermatome with a rash that forms days later. Shingles is characteristically present unilaterally.

Which of the following signs and symptoms are MOST characteristic of herpes zoster? 1. Unilateral scales along the affected dermatome followed by numbness and burning 2. Bilateral scales along the affected dermatomes followed by numbness and burning 3. Pain and paresthesia followed by a unilateral rash along the affected dermatome 4. Pain and paresthesia followed by a bilateral rash along the affected dermatomes

4. Prompted voiding 1. Continuous catheterization is usually used for patients who have a neurological deficit that directly affects urinary function (p. 917). 2. Functional exercises and breathing exercises help to retrain and strengthen the pelvic floor muscles, assisting with the weakness associated with stress incontinence. However, this intervention works best for patients who have stress incontinence, are able to know when they have to void, and do not have neurological deficits or cognitive deficits affecting their ability to control voiding. (p. 915) 3. Biofeedback can be used as an adjunct to functional exercises, but the patient must be cognitively aware to use biofeedback (p. 915). 4. Prompted voiding is used for patients who may have cognitive deficits and require reminders of regular intervals for voiding (p. 917).

Which of the following strategies for intervention would be MOST appropriate for a patient with moderate Alzheimer disease and urinary incontinence? 1. Continuous catheterization 2. Functional exercises 3. Biofeedback 4. Prompted voiding

1. Varied task practice with variable time intervals 1. Random practice provides a higher level of contextual interference that requires the individual to retrieve practice from memory stores. Research has shown superior long-term effects for random practice due to the higher cognitive processes required. (Umphred, p. 82; O'Sullivan p. 413) 2. Massed practice is preferred for individuals with fatigue issues. Although learning occurs with this type of practice schedule, the depth of cognitive processes required is typically not as high as is expected with changes in tasks and environments. (O'Sullivan, p. 411) 3. Blocked practice is practice of one task performed repeatedly without interruption from other tasks. Although this practice allows for motor learning, it is not best for long-term retention due to the lack of variability, which is a hallmark of typical, daily movement. (Umphred, p. 82; O'Sullivan, p. 413) 4. Although complex motor skills can be broken into component parts for practice, delaying practice of the integrated tasks can interfere with the transfer effects and learning. Integrated practice is best for learning, particularly with continuous movement. The daily practice of the same task may improve performance, yet may not be best for long-term retention, as it does not allow the individual to retrieve from cognitive stores. (O'Sullivan, p. 413) Patient-preferred intervals will not be the best choice, considering that optimal learning occurs with random practice schedules

Which of the following types of practice is MOST appropriate for long-term motor learning for a patient with a cerebrovascular accident? 1. Varied task practice with variable time intervals 2. Task practice of one activity for 15 minutes with 10-minute rest 3. Practice of a variety of related skills in blocks of 5 minutes 4. Partial task practice with patient-preferred time intervals

2. splint the wrists of patients with rheumatoid arthritis, because the study indicates significant differences between the control and treatment groups. 1. This response is the opposite of the correct response. The response demands that the candidate knows that p = 0.001 is a highly significant value and that the research it represents should be considered seriously in the treatment of patients with rheumatoid arthritis. 2. This research article supports the use of splinting for patients with rheumatoid arthritis, because the p value is highly significant and supports rejection of the null hypothesis (H0). Also, this study supports evidence-based practice; therefore, the physical therapist should consider splinting for patients with rheumatoid arthritis. 3. The student should recognize that a p value of 0.001 is always significant regardless of sample size. The problem with small sample sizes is the possibility of not having a great enough power of the test to pick up differences if they truly exist. Because differences were found at a highly significant level (p = 0.001), consideration of splinting is supported by the research study. 4. This response presupposes all possible treatments for rheumatoid arthritis should be considered. The candidate should recognize that a study and its parameters are identified at the beginning of the investigation and that the investigators have the latitude to determine which interventions they wish to study

a study investigating the change in pinch strength after hand splinting in patients with rheumatoid arthritis, the strength was greater in the group receiving the splint than in the group not using the splint (p = .001). Given this evidence, the physical therapist should: 1. not splint the wrists of patients with rheumatoid arthritis, because the study does not show significant differences between the control and treatment groups. 2. splint the wrists of patients with rheumatoid arthritis, because the study indicates significant differences between the control and treatment groups. 3. wait for further studies before splinting the wrists of patients with rheumatoid arthritis, because the sample size of the current study was not given. 4. wait for a pilot study comparing the current treatment group to a control group receiving anti-inflammatory medication, because the current study ignored other treatment options.


Related study sets

Taxation of Life Insurance and Annuities- Premiums and Proceeds

View Set

Florida Laws and Rules Insurance (Chapter 9)

View Set

INTG BUS POLICY/STRATEGY - Week 6

View Set